

Hello,
Dr. Batman
Hello Doctor, Welcome!
Profile

Name: Batman
Email: batman@gotham.com
EMERGENCY MEDICINE
(Total Questions - 188)Q.1. A 55 year old woman presents to A&E with the complaint of aching chest pain. She is very nervous and anxious as she thinks that this could be a heart attack. She has no family history of cardiac disease and has no other medical problems. She was given sublingual GTN which has failed to relieve her symptoms. Upon further questioning, she admits that the pain is worse when she breathes in. Her chest X-ray is reported as ‘normal’ and an ECG is performed, which also shows no significant acute changes. Her vitals are as follows: Temperature: 37.2oC Heart rate: 110 beats/minute Blood pressure: 125/85 mmHg A full blood count has been done and shows: Haemoglobin: 12.8 g/dL Platelets: 320 x 109/L White chell count: 7.8 x 109/L CRP is 10.7 mg/L What is the most likely diagnosis in this patient?
Correct Answer : C
This patient’s investigations have all returned to normal. The clue here is picking up the fact that her symptoms were not relieved by sublingual GTN.
The stem also drops another hint letting you know that she has no medical problems. She therefore does NOT have diabetes, hypertension, hypercholesterolemia, or a smoking history which are risk factors for acute coronary syndrome. Do not get thrown by the slight tachycardia in the stem as it is likely the result of anxiety or chest discomfort.
Costochondritis is the answer here and can sometimes mimic symptoms of an acute coronary syndrome. Movements of the rib cage can produce pain. Costochondritis is inflammation of the costochondral junctions of ribs or chondrosternal joints of the anterior chest wall. It is sometimes called Tietze’s syndrome, but it is not the same thing.
Tietze’s syndrome is the addition of swelling over the affected joints of the anterior chest wall.
Q.2. A 10 year old boy is rushed to Accident & Emergency after his parents found him unconscious on the kitchen floor. It is revealed that he ingested some medication belonging to his grandmother. His grandmother was unable to identify which medications are missing. On examination, the child is hypotensive with dilated pupils and dryness in the mouth. His ECG showed prolongation of the PR, QRS, and QT intervals. Which most likely medication has he taken in excess?
Correct Answer : A
The key to this question is trying to figure out what medication this child has taken and try to tie it in with his signs and symptoms.
Amitriptyline: causes drowsiness, hypothermia, hypotension, tachycardia, and dry mouth.
ECG shows arrhythmias, particularly changes in the QRS width, dilated pupils.
The classic features of digoxin toxicity are nausea, vomiting, diarrhea, abdominal pain, headache, dizziness, confusion, delirium, and vision disturbance (blurred or yellow vision). It is also associated with cardiac disturbances including irregular heartbeat, ventricular tachycardia, ventricular fibrillation, sinoatrial block, and AV block.
Metoprolol: bradycardia, hypotension, hypothermia, hypoglycemia (especially in children), and seizures.
Thiazide: hypotension, fever, frequent urination, hypotension, muscle cramps, and twitching.
Q.3. A 25 year old woman has been feeling anxious and nervous for the last few months. She also complains of palpitations and tremors. Her symptoms develop rapidly and last for a few minutes. She mentions that taking alcohol initially helped her relieve her symptoms but now this effect is wearing off and she has palpitations and tremors even after drinking alcohol. What is the most likely diagnosis?
Correct Answer : A
There is a fine line between Generalised Anxiety Disorder (GAD) and Panic attacks. They both can present similarly.
However, in this question, her symptoms develop rapidly and only last for a few minutes. This is the key phrase that you should look out for that tells you this is a Panic attacks rather than GAD.
PANIC ATTACKS : Period of intense fear characterized by symptoms that include palpitations, sweating shaking, and shortness of breath that develop rapidly.
It peaks around 10 minutes and then gradually resolves over the next 20 minutes.
One may see the extreme where the patient may have feelings of going to die from cardiac or respiratory problems or one may see the usual complaint of dizziness, circumoral paraesthesia, carpopedal spasm, and occasionally sharp or stabbing chest pain. Patients are usually tachycardic.
It is important to rule out secondary causes of tachycardia, chest pain, or shortness of breath thus investigations such as oxygen saturation, ECG, and blood glucose are important initial investigations.
If symptoms do not settle in a few minutes, consider adding on investigations such as a chest X-ray, urea and creatinine, and a full blood count.
Management : Simple breathing exercises like breathing through the nose and reassurance are all that is needed.
Q.4. A 24 year old male presents to A&E with 40% partial thickness burns. His pulse rate is 105 bpm and respiratory rate is 25 breaths per minute. His systolic blood pressure is 80 mmHg. What is the most appropriate management?
Correct Answer : B
Remember that the Parkland formula is calculated from the time of the burn rather than the time of presentation.
PARKLAND FORMULA : We use the Parkland formula to count the fluids required after burns.
This is calculated from the time of the burn rather than the time of presentation. If the presentation is delayed, fluid may need to be given more rapidly.
Fluid requirement = Body area burned (%) x Weight (kg) x 4 mL.
Give 1/2 of the total requirements in 1st 8 hours, then give 2nd half over the next 16 hours.
The area of body burn is calculated by adding the percentage of burns in each area using Wallace’s rule of 9s.
• Head and neck total for front and back: 9%
• Each upper limb total for front and back: 9%
• Thorax and abdomen front: 18%
• Thorax and abdomen back: 18%
• Perineum: 1%
• Each lower limb total for front and back: 18%
Q.5. A 6 month old boy has been brought to A&E following an apnoeic episode at home. He is now completely well but his parents are very anxious as their family friend’s child died of sudden infant death syndrome at a similar age. The parents would like to know how to perform CPR on a baby of his age. What is the most recommended technique for cardiac compressions?
Correct Answer : C
There are two options here.
Either index and middle finger of one hand or grip the chest in both hands in such a way that two thumbs can press on the lower third of the sternum.
Q.6. A 35 year old woman drinks 2 litres of vodka a day. She is well known to the hospital as she is frequently brought into accident and emergency department drunk with minor injuries. She now wants to stop drinking alcohol and is willing to seek help for her alcohol dependence. However, her main concern is that she has no support and lacks encouragement. What would you do?
Correct Answer : B
The patient does not require admission to the hospital for detoxification but does require social support and encouragement. Social services have access to some different services including outpatient detoxification, group therapy sessions, and supportive networks.
In addition, women who are alcohol dependent are at risk of abuse, exploitation, and unwanted pregnancy.
Social services can coordinate a multi-disciplinary approach and create a package of care to facilitate safe detoxification.
Cognitive behavioural therapy (CBT) is a talking therapy aimed at changing the way a person thinks and behaves.
Although at first, it appears CBT would be appropriate for this patient, it must be noted that she has no support network and no package of care in place for her safety. There is no indication to refer to psychology.
Q.7. An 18 year old, previously well student in his first year at university, was brought into the emergency department in an agitated, deluded and disoriented state. What is the most probable reason for his condition?
Correct Answer : A
Young age and first year in university are likely to point towards drug toxicity.
Q.8. A 20 year old man presents to A&E after having severe injuries from a road traffic accident. On presentation he is breathless and has severe chest pain. Paradoxical respiration with shortness of breath is seen. His systolic blood pressure is 85 mmHg, respiratory rate is 25 breaths/min, oxygen saturation of 88% and his pulse rate is 110/min. What is the most appropriate initial action?
Correct Answer : B
In this question, the examiners want you to know the basics of life-threatening emergencies.
ABC – airway, breathing, and circulation should always be addressed first. In reality, oxygen by mask, securing venous access, and analgesia would all be done simultaneously.
But for this exam, we should know the steps, thus, securing airways and giving oxygen would come before anything else.
Remember ABC!
FLAIL CHEST : A flail chest occurs as a result of trauma to the chest, leading to at least 3 ribs becoming fractured or broken, close together, with pieces of bone detaching from the chest wall.
These segments of bone start to move independently of the chest wall and in the opposite direction because of lung pressure. The result is a “paradoxical respiration”.
Aetiology :
• Fall (for example, off a bicycle or a horse)
• Blunt trauma to the chest
• Car Accident
Diagnosis: Paradoxical respiration with shortness of breath and chest pain in a patient who has just had blunt chest trauma raises the suspicion of a flail chest.
The diagnosis is usually clinical but with the help of a chest X-ray which would demonstrate rib fractures.
Management:
1. High flow oxygen
2. Analgesia such as paracetamol / NSAIDS / Opiates + splinting of injury
3. Intubation / mechanical ventilation - if worsening RR.
Q.9. A 54 year old man has just had a cardiac arrest. He is currently being managed in the Intensive Care Unit (ICU). He is on a ventilator. His vitals were taken and are as follows: Pulse 120 beats/minute Blood pressure 100/50 mmHg An arterial blood gas was also done for this patient and the results are as follows: pH 7.03 pCO2 9.8 kPa PaO2 12 kPa HCO3 18 mmol/L. What is the most appropriate next step in management of this patient?
Correct Answer : A
This patient’s pH is abnormal his PaCO2 is abnormal (it is high) AND his HCO3 is abnormal (it is low) therefore, we can conclude that this patient has a mixed acidosis picture.
This is the simplest and best time-saving way to interpret ABG scenarios in the exam. You do not even have to look at the PaO2 value. Increasing ventilation would immediately increase the release of carbon dioxide and help resolve the acidosis.
Q.10. A 21 year old man has been found unconscious in an alleyway with a respiratory rate of 5 breaths/minute and a pulse of 55 beats/minute. His pupils are constricted. What is most appropriate management?
Correct Answer : B
This is a classic case of opioid overdose. The word “alleyway” already answers away.
Respiratory depression, bradycardia, and miotic pupils support the clinical picture. Naloxone is used to block the effects of opioids, especially in overdose.
ILLICIT DRUGS AND INTOXICATION :
Heroin :
• Respiratory depression
• Pinpoint pupils
• Mild hypotension
• Bradycardia
• Weak pulse
• Constipation
Cocaine :
• Increase respiratory rate and depth
• Mydriasis
• Hypertension
• Tachycardia
• Arrhythmias
• Sweating
• Nausea
• Twitching of small muscles
• Elevated body temperature
• Restlessness and agitation
Ecstasy :
• Uncontrolled body movements
• Hyperthermia
• Increase in blood pressure
• Insomnia
• Trismus
• Tachycardia
• Increase in respiratory rate
LSD (Lysergic acid diethylamide) :
• Mydriasis
• Flushing and sweating
• Diarrhoea
• Paraesthesias
• Hyperactive reflexes
• Delusions and hallucinations
• Remember the effects of LSD are primarily on the 5 senses:
Sight – Colours become vivid,
Hearing – Hearing becomes more acute,
Touch – Touch is intensified,
Smell – May even smell colors,
Taste – tasting things that aren’t there
Q.11. A 9 month old child aspirated a foreign object which was removed at hospital. The child is now fine. His parents would like to know what they should do should this occur again. What is the most appropriate advice to give them?
Correct Answer : C
CHOKING AND FOREIGN BODY AIRWAY OBSTRUCTION IN INFANTS :
• In a seated position, support the infant in a head-downwards, prone position to let gravity aid the removal of the foreign body
• Deliver up to five sharp blows with the heel of your hand to the middle of the back
• After five unsuccessful back blows, use chest thrusts: turn the infant into a supine position and deliver five chest thrusts. These are similar to chest compressions for CPR, but sharper and delivered at a slower rate.
Q.12. A 49 year old woman presents to the Emergency Department with a productive cough of green sputum. She feels unwell, feverish and lethargic. On examination, bronchial breathing is heard at her right base. She has a respiratory rate of 27 breaths/minute, oxygen saturation of 90% on room air, pulse rate of 130 beats/minute and a blood pressure of 85/40 mmHg. What is the next most appropriate action?

Correct Answer : B
She is septic from pneumonia. Sepsis six would need to be performed urgently within the hour. Among the sepsis six is to give intravenous fluids. Oral antibiotics would be inappropriate.
We need a broad-spectrum antibiotic in the bloodstream as soon as possible thus it needs to be given intravenously. Chest X-ray would need to be performed but can be done after performing the sepsis six and stabilizing the patient.
SEPSIS :
Sepsis is defined as a life-threatening organ dysfunction caused by a dysregulated host response to infection.
'Red Flags' in sepsis -
• Systolic blood pressure < 90 mmHg
• Heart rate >130 beats/minute
• Oxygen saturations < 91%
• Respiratory rate >25 breaths/minute
• Responds only to voice or pai or unresponsive
• Lactate > 2.0 mmol
• `Reduced urine output.
If 1 or more RED FLAGS are present :
Complete the SEPSIS SIX within 60 minutes. Sepsis Six - Take 3, Give 3
• Take 3 : Blood culture + FBC, urea and electrolytes + clotting & lactate. Start monitoring urine output.
• Give 3 : High flow oxygen, Intravenous fluid challenge, Intravenous antibiotics.
Septic shock is defined as persistently low blood pressure which has failed to respond to the administration of intravenous fluids.
Q.13. A 44 year old man with a history of alcohol dependency presents with confusion. He responds poorly to questions and is seen to be irritable. He is unsteady and has uncoordinated walking. His blood glucose is 3.5 mmol/L. He has no other relevant medical history. What is the most appropriate immediate action?
Correct Answer : D
It is well known that chronic alcoholics are at high risk for being deficient in vitamin B1 (thiamine), which is known to put the patient at an increased risk for Wernicke-Korsakoff Syndrome and cerebellar degeneration. Thiamine should be given priority in this stem before administering glucose.
Q.14. A 5 year old boy with a febrile convulstion lasting eight minutes. He has been given IV lorazepam to control his seizures. What is the most likely side effect of IV lorazepam that is potentially life threatening?
Correct Answer : B
Lorazepam is a benzodiazepine. Respiratory depression is a known effect of benzodiazepine overdose. Amnesia can occur as well, but it will not be life-threatening.
Side effects of benzodiazepines include:
• sedation
• cognitive impairment
• respiratory depression
• hypotension
• anterograde amnesia.
Q.15. A 23 year old woman has been found unconscious by her partner. There are several packets of paracetamol and an empty bottle of vodka alongside her. When she comes to the Emergency Department, she is confused and unable to estimate when she took the tablets. Her Glasgow Coma Scale (GCS) score is 14/15. Which is the most appropriate next step?
Correct Answer : C
In cases like this where a serious overdose is suspected, give paracetamol antidote N-acetylcysteine immediately. The amount of paracetamol taken is known in this question. N-acetylcysteine infusion should be started if there is doubt over the time of paracetamol ingestion, regardless of the plasma paracetamol concentration.
The 4-hour figure refers to the time after ingestion at which plasma levels can be interpreted. If levels turn out to be below the treatment line, N-acetylcysteine can be stopped.
Q.16. A 28 year old woman, with a background of chronic alcoholism presents with bright-red haematemesis and abdominal pains. On examination, she has cool extremities and is seen to be confused. Her blood pressure is 85/60 mmHg and her heart rate is 120 beats/minute. What is the most appropriate next step in management?
Correct Answer : C
Key points to remember for the management of acute upper gastrointestinal bleeding possibly due to varices:
• Always start with fluid resuscitation
• Endoscopy should be undertaken immediately after resuscitation for unstable patients with severe acute upper gastrointestinal bleeding. This allows the bleeding point to be visualized and treated.
* Band ligation is the first choice treatment for all patients with upper gastrointestinal bleeding from oesophageal varices.
• Administer terlipressin at presentation (2 mg intravenously repeated every 4 to 6 hours). Treatment should be stopped after definitive haemostasis has been achieved.
• Avoid omeprazole acutely unless the patient has known peptic ulcer disease. PPIs should not be used before diagnosis by endoscopy
• If INR is prolonged, administer vitamin K
• Administer platelet transfusions to patients who are actively bleeding and have a low platelet count.
• TIPS should be considered only if bleeding from oesophageal varices is not controlled by band ligation.
• Balloon tamponade is only used as a salvage procedure in a patient with massive haemorrhage, at risk of death.
• Administer prophylactic antibiotics e.g. ciprofloxacin or cephalosporin. This can be administered at the presentation. The first four key points are the most important to remember and will get you through the exam.
Q.17. A 33 year old man has severe burns on his chest and arms after his shirt caught on fire during a barbecue event. Which of the following formulas is used for calculating fluids for burn patients?
Correct Answer : C
Please see Q-4
Q.18. A 32 year old man presents to the Emergency Department after a motorcycle crash. The patient has bruises around the left orbital area. His Glasgow coma scale (GCS) is 14 on initial assessment in the department. On examination, an alcoholic breath is noted. 2 hours after the initial assessment, his GCS drops to 8. What is the most important initial investigation?
Correct Answer : B
His GCS is less than 15 at 2 hours after the injury on assessment in the emergency department and he has signs of basal skull fracture on top of the additional risk factor which is a dangerous mechanism of injury. He needs an urgent CT head.
Q.19. A 12 year old girl when playing in the garden accidentally stepped on a hive and was bitten several times. She has numerous wheals on her body and complains of severe itching which is worsening in the last few hours. What is the most appropriate management?
Correct Answer : A
This girl is suffering from urticaria. Oral antihistamines would be indicated.
Since this is an allergic reaction, the only two reasonable options are A and C in this question. However, in the context of allergic reactions, IM adrenaline should only be used in an anaphylactic shock whereby the patient would have breathing difficulties.
The following are the indications of adrenaline in anaphylaxis:
1. Hoarseness of voice
2. Wheeze
3. Shortness of breath
4. Shock
5. Stridor
6. Swelling of the tongue and cheek
7. Facial swelling
If you see an urticaria-like allergic reaction, with none of the following indications stated above for adrenaline, then pick the oral antihistamine as the answer.
Q.20. A 47 year old man had a road traffic accident and has presented to A&E with multiple injuries. On physical examination, perineal bruising was noticed. A pelvic fracture has been confirmed. He has not passed urine since the accident which was 7 hours ago. What is the most appropriate next course of action?
Correct Answer : C
The pelvic fracture and perineal bruising indicate that there may be a urethral injury. If you suspect a urethral injury, do not attempt to perform urethral catheterization.
In the exam, the option of suprapubic catheterization is usually given. Pick that. Posterior urethral tears are often associated with pelvic fractures. Urethral injury may also result from blows to the perineum.
Look for perineal bruising and blood at the external urethral meatus and perform a rectal examination.
If urethral injury is suspected, do not attempt urethral catheterization, but refer urgently to the urology team. Some urologists perform a retrograde urethrogram to assess urethral injury, but many prefer suprapubic catheterization and subsequent imaging.
Q.21. A 4 year old child playing with toys unattended suddenly develops breathlessness and stridor and is rushed into the hospital by his father. The child is drooling and unable to swallow. What is the best investigation likely to lead to a diagnosis?
Correct Answer : D
Breathlessness and stridor in a child playing with toys is most likely due to aspiration of a foreign body (e.g. a part of the toy) for which indirect laryngoscopy and/or fiber-optic examination of the pharynx would provide a diagnosis. Remember the ingestion of foreign bodies is most commonly a problem in young children aged 6 months to 5 years.
Q.22. A 32 year old man who was rescued from a building on fire presents to A&E unconscious. He has no evidence of burns or any external injuries. His heart rate is 135 beats per minute and his blood pressure is 85/55 mmHg. What is the most appropriate management?
Correct Answer : C
As this patient is unconscious, we need to intubate and provide IPPV on 100% O2.
For a conscious patient use a tight-fitting mask with an O2 reservoir. The decision for hyperbaric oxygen therapy (HBOT) is a difficult one and there are many debates about the added value provided by hyperbaric oxygen.
Q.23. A 61 year old man was brought to the Emergency Department by ambulance after a road traffic accident. His blood pressure is 80/40 mmHg and respiratory rate is 39 breaths/minute. His oxygen saturations are 88%. He has bruises on his chest. His chest wall is not moving symmetrically. He has labored breathing, accessory muscle use and fluctuates in and out of consciousness. What is the most appropriate initial action?
Correct Answer : D
In this question, the examiners want you to know the basics of life-threatening emergencies.
ABC – airway, breathing, and circulation should always be addressed first. The patient is in shock.
In reality, high-flow oxygen, securing venous access, and analgesia would all be done simultaneously. But for this exam, we should know the steps.
Thus, securing airways and giving oxygen would come before anything else. Positive pressure ventilation would suffice in most cases of flail chest however this patient needs early intubation as he is acutely deteriorating.
A flail chest like in this stem may require intubation with a double lumen tracheal tube. In a double-lumen endotracheal tube, each lumen may be connected to a different ventilator. With a flail chest, one side of the chest is affected more than the other, so each lung requires different pressures and flows to adequately ventilate.
MANAGEMENT OF SHOCK :
Investigation and treatment should occur simultaneously
Address the priorities
ABC
High flow O2 by mask
Secure adequate venous access and take necessary blood investigation
Resuscitation is usually started with crystalloids, such as normal saline.
Q.24. A 76 year old woman has become tired and confused following an influenza like illness. She is vomiting and with abdominal pain. She is also breathless with signs of consolidation of the left lung base. Her temperature is 39.0oC. She has a blood pressure of 80/60 mmHg. A blood count showed: Haemoglobin 120 g/L White cell count 19.1 x 109/L Platelets 90 x 109/L. What is the most likely diagnosis?
Correct Answer : C
TOXIC SHOCK SYNDROME :
Symptoms of toxic shock syndrome vary depending on the underlying cause. Toxic shock syndrome resulting from infection with the bacterium Staphylococcus aureus typically manifests in otherwise healthy individuals with high fever, accompanied by low blood pressure, malaise, and confusion, which can rapidly progress to stupor, coma, and multiple organ failure.
The characteristic rash, often seen early in the course of illness, resembles a sunburn and can involve any region of the body, including the lips, mouth, eyes, palms, and soles.
In patients who survive the initial phase of the infection, the rash desquamates, or peels off, after 10–14 days.
WBC is usually increased and platelets are decreased.
Signs and symptoms of toxic shock syndrome:
• Body temperature > 38.9 °C
• Systolic blood pressure < 90 mmHg
• Diffuse macular erythroderma
• Desquamation (especially of the palms and soles) 1–2 weeks after onset.
Involvement of three or more organ systems:
• Gastrointestinal - vomiting, diarrhea
• Muscular: severe myalgia or creatine phosphokinase level elevation
• Mucous membrane hyperemia
• Kidney failure
• Liver inflammation.
Q.25. A 9 year old child was admitted following a road traffic accident. On admission his initial GCS was 15. His GCS dropped to 13 during the night. What is the most appropriate management?
Correct Answer : D
A CT head scan should be performed if GCS < 15 at 2 hours post-injury in children.
Q.26. A 32 year old woman starts bleeding profusely in theater during an elective caesarean section. She had a spinal block and was awake throughout the procedure. She is now found to be unconscious on the theatre table. Her blood pressure has dropped to 70/40 mmHg. What is the most likely diagnosis?
Correct Answer : A
Primary haemorrhage occurs at the time of surgery. It is bleeding immediately after an injury or operation.
Treatment involves replacing blood. If severe, return to theatre for adequate haemostasis.
The other options given are less likely :
Reactionary haemorrhage - Occurs within the first 24 hours and it is usually due to venous bleeding and is usually caused by slipping of ligatures, dislodgement of clots, and patient warming up after surgery causing vasodilation, on top of normalization of blood pressure.
Secondary haemorrhage - Usually caused by necrosis of an area of the blood vessel, related to the previous repair, and is often precipitated by a wound infection. It occurs 1 to 2 weeks post-op.
Septic shock - There are no signs or symptoms of infection to indicate sepsis.
Q.27. A 31 year old man was knocked down during a fight in the waiting room of the emergency department. He is now unconscious and unresponsive. What is the most important first action?
Correct Answer : C
The patient in the stem was knocked unconscious - there is nothing to indicate that he has stopped breathing and so, CPR is unnecessary.
Primary survey – this follows the ABCDE principle
• Airway maintenance with cervical spine protection
• Breathing and ventilation
• Circulation with haemorrhage control
• Disability with assessment of neurological status
• Exposure / Environmental control.
Q.28. A 55 year old lady comes in with severe haemetemesis. She is anxious and aggressive. Her medical history includes liver disease. Her INR is 9; heart rate is 110 bpm; Systolic BP is 110 mmHg; SpO2 is 94%. What is the most appropriate management?
Correct Answer : A
The most appropriate management here would be FFP as she has a history of liver disease and a high INR.
Q.29. A 33 year old man is stabbed with a knife in his thigh. He has tried to use a towel to stop the bleeding but has bled so much that the towel is now soaked with blood. His blood pressure is 85/50 mmHg, pulse rate is 132 beats/minute and respiratory rate is 31 breaths/minute. What percentage of circulatory blood did he lose?
Correct Answer : C
STAGES OF HYPOVOLAEMIC SHOCK :
For adults, the clinical staging relating to loss of blood volume can be classified as:
Stage 1: Blood loss - 10-15%
Stage 2: Blood loss - 15-30%
Stage 3: Blood loss - 30-40%
Stage 4: Blood loss - Over 40%
You would need to memorize the amount of blood loss in each stage e.g. 30-40% in stage 3.
Q.30. A 30 year old patient is brought to the emergency department after a road traffic accident. He has multiple bruises on his chest and an area of paradoxical rib movement. A chest X-ray shows a right-sided pulmonary contusion. He has a pulse of 129 beats/minute, a blood pressure of 100/70 mmHg and a respiratory rate of 33 breaths/minute. He has severe chest pain and is dyspnoeic. What is the best management for his condition?
Correct Answer : B
In cases of paradoxical rib movement i.e, Flail chest, the most important aspect of treatment is to treat the pain due to rib fractures. Because the pain will lead to hyperventilation and exaggerated movement of rib cage, and we do not want that to occur.
Q.31. A 21 year old man, who was heavily drinking a few hours ago at an evening party presents to Accidents & Emergency with a history of vomiting repeatedly during the night. He vomited fresh blood an hour ago. Since then, he has had no further bleeding. He denies any melena. He has a heart rate of 70 beats/minute, a blood pressure of 120/80 mmHg, a respiratory rate of 15 breaths/minute. His blood results show: Urea 4 mmol/L Haemoglobin 153 g/L. What is the most appropriate management?
Correct Answer : D
This patient has Mallory-Weiss tears from drinking too much alcohol. As he is vitally stable, and his haemoglobin count is high.
The best plan is to discharge him home with advice. This is a very high-yield question. Occasionally, the option of “discharge with advice” is not there, in which case they would provide other correct options such as
• Repeat FBC
• Observe vital signs for deterioration
Q.32. A 23 year old single male was brought to emergency department by his father exhausted and frightened. His father tells you that his son, who was previously healthy, had, for no apparent reason, a sudden attack of fear, dizziness, sweating, palpitations and the feeling that his heart is going to stop beating. The symptoms started to decrease gradually after about 10 minutes. Which is the most likely diagnosis?
Correct Answer : C
Panic attacks usually have a fast onset and resolve quickly as well. They usually take about 10 minutes to peak and then followed by resolution over the next 20 minutes. Some attacks may be situational such as in specific scenarios where attacks have occurred previously whilst other attacks may be spontaneous and described as “out of the blue” which is seen in this stem.
Q.33. A 21 year old lady after a heavy bout of drinking last night comes to the emergency department with vomiting blood, feeling dizzy, and having intense abdominal pain. On examination, her limbs feel cold. After initial resuscitation with oxygen and fluids, she still continues to bleed and continues to vomit blood. She has a pulse of 130 beats/minute and blood pressure of 85.58 mmHg. What is the next best step?
Correct Answer : B
This is a classic presentation of Mallory-Weiss syndrome.
It is characterized by upper gastrointestinal bleeding (UGIB) from mucosal lacerations in the upper gastrointestinal tract, usually at the gastro-oesophageal junction or gastric cardia. Excessive alcohol ingestion is one of the main causes as prolonged or forceful bouts of vomiting can cause a tear in the upper gastrointestinal tract.
Resuscitation is a priority - maintain the airway, provide high-flow oxygen, and correct fluid losses by giving IV fluids.
Intravenous blood can also be given in severe cases. Haemodynamically unstable patients like those in this stem should have an endoscopy immediately after resuscitation. It is the primary diagnostic investigation and can be used to stop the bleeding.
Note, that proton pump inhibitor (PPI) use is not recommended prior to diagnosis by endoscopy.
Q.34. A 32 year old man presents to A& E in hypovolaemic shock immediately following a road traffic accident. He is tachycardic, sweating and anxious. He was also noted to have bruises on his abdomen. What mechanism explains the manifested symptoms?
Correct Answer : A
You should never get a pathophysiology question wrong for the exam. This man has experienced major trauma. The bruises on his abdomen point you toward a possible intra-abdominal bleed.
Here is the mechanism for hypovolaemic shock: Early changes in Loss of blood volume stretch receptors in the atria and baroreceptors in the aorta become activated.
Q.35. A 55 year old women recovering from a surgery for a toxic goiter is found to be cyanosed in the recovery room. Her neck is tense and her blood pressureis 85/45 mmHg. There is blood oozing from the drain. What is the most likely diagnosis?
Correct Answer : C
Reactionary Haemorrhage occurs within the first 24 hours following trauma/surgery and it is usually caused by slipping of ligatures, dislodgement of clots, patient warming up after surgery, causing vasodilation, on top of normalization of blood pressure.
Haemostasis appears secure until blood pressure rises to normal then bleeding begins. Treatment usually involves replacing blood and re-exploring the wound.
Hypotension, tense neck, and blood oozing from the drain tell us that the patient is probably bleeding. As the patient is still in the recovery room, the surgery is likely recent (less than 24 hours) thus the diagnosis of reactionary haemorrhage.
Q.36. A 26 year old man presents to the Emergency Department with the complaint of a 24 hour history of epigastric pain and haematemesis. He had ingested an unknown drug four days ago, but he cannot remember the name of the drug or the amount of the drug that he took at the time because he had been drunk. He has no past medical history of note and takes no chronic medication. He indulges in social drinking occasionally. Blood tests were done, and the results are as follows: Creatinine 230 micromol/L (70-150 micromol/L) Bilirubin 20 micromol/L (3-17 micromol/L) Alanine transferase (ALT) 1880 U/L (5-35 U/L) Aspartate transaminase (AST) 3150 U/L (5-35 U/L) Alkaline phosphatase (ALP) 215 U/L (30-150 U/L) Gamma glutamyl transferase (gamma GT) 80 U/L (8-60 U/L) Prothrombin time (PT) 20 seconds (10-14 secs) He has a pulse rate of 128 beats/minute, blood pressure of 100/60 mmHg, respiratory rate of 25 breaths/minute and a temperature of 37.1oC. What most likely drug did this patient ingest?
Correct Answer : B
The raised level of serum creatinine, bilirubin, and liver enzymes as well as the increased prothrombin time indicates that this patient has taken an overdose of paracetamol. Paracetamol is an antipyretic and analgesic. Patients are usually asymptomatic for the first 24 hours or have nonspecific abdominal symptoms. Hepatic necrosis begins to develop after 24 hours and can progress to acute liver failure.
The liver damage is not usually detectable by routine liver function tests until at least 18 hours after ingestion of paracetamol.
Alanine transferase (ALT), prothrombin time usually peaks 72 to 96 hours after ingestion.
Amitriptyline is a tricyclic antidepressant. TCA’s taken in large amounts can cause excess sedation, and sympathomimetic effects - tachycardia, sweating, dilated pupils.
Aspirin is a potent antiplatelet agent as well as an antipyretic, analgesic, and anti-inflammatory agent. An overdose of aspirin causes ringing in the ears, nausea, abdominal pain, and tachypnoea.
Organophosphates are usually the active ingredients in insecticides. Symptoms include increased saliva and tear production, diarrhoea, vomiting, small pupils, sweating, muscle tremors, and confusion.
Q.37. A 31 year old man was involved in a road traffic accident and has severe pain at the right outer upper thigh and groin. There is clear deformity of the hip and shortening of the right leg. A femoral shaft fracture is suspected. His blood pressure is 100/70 mmHg. He has a heart rate of 90 beats/minute and a respiratory rate of 19 breaths/minute. He is saturating at 97% at room air. What is the most appropriate next action?
Correct Answer : C
In any trauma-associated emergency, including fractures of the shaft of the femur, we should apply Advanced Trauma Life Support principles and attend to the femur only once we are happy with our ABCDEs. As this patient is clinically stable, we should move on to attend to the femur by splinting it.
Intravenous fluids, full blood count, and cross match would also be performed but since in this question, he is clinically stable, we should perform a Thomas’ splint first. Putting a splint will improve alignment and reduce ongoing blood loss.
Bloss loss in a femur shaft fracture is alarming as blood loss is significant. In reality, you would have more than one doctor and health care professional in this scene, thus one would be putting an IV cannula and sending blood while the others would be splinting the leg. Imaging is important but can be done at a later time.
Q.38. A 44 year old female comes to the Emergency Department with complaints of the worst headache of her life which started two days ago. The pain is at the back of her head and the maximum pain was experienced 8 hours ago after which her pain has slowly been resolving. She vomited twice yesterday. She has no history of migraines and is usually fit and healthy. She was assessed by the triage nurse and had a Glasgow Coma Scale of 15/15, equal and reactive pupils and no neurological deficit. Whilst waiting for the doctor to assess her, she becomes unresponsive and a quick assessment shows a Glasgow Coma Scale of 3/15, right pupil larger than the left pupil and response sluggish to light. Her blood pressure is 155/80 mmHg, heart rate is 100 beats/minute and respiratory rate is 22 breaths/minute. What is the most appropriate next step in management?
Correct Answer : C
This is a medical emergency. An emergency medical crash call would be appropriate of which the two important teams that should be involved are the anaesthetist and the medical team. The anaesthetist needs to urgently review this patient as intubation would be the first part of management to secure her airways.
Airways, breathing, and circulation would need to be addressed quickly and the patient should be moved into a recovery position whilst waiting for the anesthetist to arrive.
An urgent CT head scan is also important as the likely diagnosis here is a subarachnoid haemorrhage however, the patient would need to be stabilized first by the anaesthetic team prior to moving her to the CT scan room.
The neurosurgical team would also need to be involved but they should only be called once the CT images are made available which shows a brain haemorrhage.
Antihypertensives such as nimodipine are part of the management and are often used by neurosurgeons following aneurysmal subarachnoid haemorrhage however there should be evidence of a subarachnoid on CT head first.
Q.39. A 24 year oldman is brought into the Emergency Department by his friend who drops him off and leaves before speaking to any staff member. He has a Glasgow Coma Scale of 9. His heart rate is 50 beats/minute, respiratory rate is 9 breaths/minute and blood pressure is 110/70 mmHg. On examination, there are needle marks on his forearm and both his pupil diameters are measured at 2 mm. What is the most appropriate management?

Correct Answer : B
A low respiratory rate, constricted pupils, and semiconscious are features of opioid poisoning.
Naloxone is a specific antagonist for opioids and is used especially in opioid overdose to reverse coma and respiratory depression. When given intravenously, it can begin to work within 2 minutes.
Q.40. A 33 year old woman was painting her room when she developed a severe headache, vomited and fell off the stool. She now feels well and has a Glasgow Coma Scale of 15. On examination, there is a small bruise on her forehead. What is the next appropriate investigation to identify the cause?
Correct Answer : C
The woman is likely to be suffering from Carbon Monoxide (CO) poisoning caused by inhalation of Methylene Chloride from the paint fumes.
The symptoms of a severe headache, vomiting, and dizziness are suggestive of CO poisoning.
Carboxyhaemolglobin levels are necessary to confirm the diagnosis.
Treatment is with 100% oxygen administered.
Q.41. An 18 month old boy pulled over a cup of hot tea that was on a high table. The hot liquid splashed over him and he now presents with a 6% partial thickness burn on his chest. What is best treatment for him?
Correct Answer : A
Any child with more than 10% of the total body surface area burned requires fluid replacement. Here there is only 6% partial thickness burns.
Q.42. A 25 year old man has been stabbed in the right-hand side of his abdomen with a small knife. He presents with severe pain in his right upper quadrant with guarding. IV fluidsare being administered. He is very anxious and agitated. His temperature is 36.5oC, heart rate 120 bpm, BP 85/55 mmHg, SaO2 97% on 10L oxygen. The A&E doctor thinks his liever might have been damaged in the attack and calls the surgeons to assess him. Which is the most appropriate initial management?
Correct Answer : C
Cross-match is the most appropriate as he may need a blood transfusion. If IV fluids have not been started and the option for IV fluids is given in this question, then pick IV fluids.
This would be the first initial step together with oxygen.
The other options given in this question are less likely: Fresh frozen plasma is used in replacement of isolated factor deficiency, reversal of warfarin effect, massive blood transfusion and treatment of thrombotic thrombocytopenic purpura and is not indicated here in this question.
Liver enzymes : Have no role in acute management.
Immediate laparotomy : This may well be the last step after all investigations are carried out.
Q.43. A 24 year old male presents to A&E with 40% partial thickness burns after having been in a house fire. His pulse rate is 115 bpm and respiratory rate is 29 breaths per minute. His systolic blood pressure is 80 mmHg. What is the most appropriate management?
Correct Answer : D
These are the steps that you need to know for the treatment of burns.
Note the order :
1. Large-calibre intravenous lines must be established immediately in a peripheral vein.
Any adult with burns affecting more than 15% of the total body surface area burned or a child with more than 10% of the total body surface area burned requires fluid replacement calculated from the time of the burn.
2. Ensure adequate analgesia: strong opiates should be used
3. Prevent hypothermia.
Q.44. A 34 year old man was involved in a road traffic accident. Whilst in the ambulance his Glasgow Coma Scale deteriorates from a score of 13 to 9. His respiratory rate increases from 30 to 48 breaths/minutes. What is the most appropriate management?
Correct Answer : B
He is in the ambulance. His oxygen demand has increased due to an increased respiratory rate.
100% oxygen would be a suitable option as part of the initial management. One would not do a needle thoracentesis unless there were clinical features of a tension pneumothorax like a deviated trachea.
Q.45. A 34 year old man had a car crash and is being observed in the emergency department. He is slowly deteriorating and his GCS has fallen from 13 to 7. What is the most appropriate next step in management?
Correct Answer : A
Intubate and ventilate all patients with GCS 8 or less. This should be done first before anything else.
Q.46. A 24 year old man comes into the emergency department with partial thickness burns all over his body from a house fire. He has a persistent cough and stridor. There are deep neck burns, carbonaceous sputum and spot in the mouth and oedema of the oropharynx. What is the most appropriate management?
Correct Answer : C
After major burns, if there is any evidence of impending airway obstruction (stridor, oropharyngeal swelling, call for senior ED help and a senior anaesthetist immediately. Urgent general anaesthesia and tracheal intubation may be life-saving. Airway, breathing, and circulation are always in this order.
Smoke inhalation injury is a common cause of death in burn victims. Initial assessment may reveal no evidence of injury, but laryngeal oedema can develop suddenly and unexpectedly thus early intubation is warranted if there is evidence of inhalation injury.
Signs and symptoms of smoke inhalation injury :
• Persistent cough
• Stridor
• Wheezing
• Black sputum suggests excessive exposure to soot
• Use of accessory muscles for respiration
• Blistering or edema of the oropharynx
• Hypoxia or hypercapnia.
Q.47. An employer sent his worker to the emergency department after having hit his head on a heavy machine. He did not lose consciousness and is not vomiting, however he does not remember the events leading up to the injury. A clinical examination including a neurological assessment is normal. What is the most likely investigations you would do?
Correct Answer : B
This patient requires a computed tomography head within 8 hours as per guidelines. There is evidence of retrograde amnesia and also a dangerous mechanism of injury.
Q.48. A 21 year old girl looking unkempt, comes to the hospital asking for painkillers for her abdominal pain. She is agitated and looks malnourished. She is also sweating, shivering and complains of joint pain. What is the most likely substance misuse?
Correct Answer : D
Agitation, nervousness, abdominal cramps, sweating, shivering, and arthralgia are all features of heroin withdrawal. The opiates are a group of chemicals that are widely abused for their euphoriant and anxiolytic properties. Heroin is the most frequently abused opiate.
Heroin is most commonly consumed by smoking, but there are so many methods to consume heroin which include taking it orally, snorting it, and through parenteral administrations such as intravenous, intramuscular, or subcutaneous routes.
Q.49. A 24 year old woman has been brought to the emergency department having taken 36 tablets of paracetamol following an argument with her partner. She weighs 60 kg. She has no previous psychiatric history and is physically well. What is the most appropriate management?
Correct Answer : C
36 tablets of paracetamol is 18 g. Remember, 1 tablet is 500 mg.
Generally, if one takes more than 24 tablets (12 g), we admit Paracetamol poisoning is always dealt with in the medical ward.
Q.50. A 20 year old man presents to A&E after having severe injuries from a road traffic accident. On presentation he is breathless and has severe chest pain. His systolic blood pressure is 70 mmHg and his pulse rate is 130 beats/minute. What is the most appropriate initial action?
Correct Answer : D
In this question, the examiners want you to know the basics of life-threatening emergencies.
ABC – airway, breathing, and circulation should always be addressed first. This patient is in shock. In reality, high-flow oxygen, securing venous access, and analgesia would all be done simultaneously.
Securing airways and giving oxygen would come before anything else.
Management of Shock :
Investigation and treatment should occur simultaneously.
• Get senior help immediately
• Address the priorities - ABC
• Give high flow O2 by mask
• Secure adequate venous access and take necessary blood investigation
• Monitor vital signs, including pulse, BP, SpO2, respiratory rate
• Insert a urinary catheter and monitor urine output hourly
• For shock associated with effective circulating blood volume, give IV crystalloid (0.9% saline) 20 ml/kg as bolus.
Q.51. A 27 year old man presents to emergency department after a road traffic accident where his right foot was stuck under a truck for several hours. On examination, his right foot is swollen and tender. Sensation is reduced between the space of the 3rd metatarsal and big toe. His dorsalis pedis pulse is not felt. What is the most likely diagnosis?
Correct Answer : A
The diagnosis here is compartment syndrome given the history of prolonged limb compression. The reason the dorsal pedis pulse is not felt is because arterial blood flow is reduced when pressure exceeds systolic blood pressure. Note that sensory deficit may occur in the distribution of nerves passing through the compartment which explains the reduced sensation in this stem. Muscle tenderness and swelling are also a part of the clinical features seen in compartment syndrome.
Compartment syndrome is a painful and potentially serious condition caused by bleeding or swelling within an enclosed bundle of muscles (a muscle ‘compartment”).
Acute compartment syndrome occurs after a traumatic injury such as a car crash. The trauma causes severe high pressure in the compartment which results in insufficient blood supply to muscles and nerves.
Acute compartment syndrome is a medical emergency that requires surgery to correct. If untreated, the lack of blood supply leads to permanent muscle and nerve damage and can result in the loss of function of the limb.
Q.52. A 33 year old woman has recurring tightness in her chest accompanied by palpitations and sweating. These episodes occur several times a week and are associated with increased respiratory rate and tingling and numbness around the mouth and fingers. What is the most likely diagnosis?
Correct Answer : D
In panic attacks, perioral paresthesia, tingling, and numbness in the hands can occur due to hyperventilation and CO2 washout leading to low ionic calcium. The description of tightening with palpitations fits with panic attacks.
Q.53. An 18 year old female was brought into the ED after ingestion of 28 paracetamol tablets after breaking up with her boyfriend. She came in confused and unwell. She was admitted in the medical ward and N-Acetylcysteine was given. 24 hours later, her laboratory results show a normal FBC, an arterial pH of 7.1, prothrombin time of 17 seconds and creatinine of 255 micromol/L. She is still confused and lethargic. What is the most appropriate management? Normal Lab values: Creatinine 70-150 micromol/L pH 7.35-7.45 Prothrombin time (PT): 11-14 sec.
Correct Answer : A
Her arterial pH is 7.1 which is an indication of liver transplantation; criteria for liver transplantation. Arterial pH < 7.3, 24 hours after ingestion or all of the following:
• Prothrombin time > 100 seconds
• Creatinine > 300 micromol/L
• Grade III or IV encephalopathy
Generally, if one takes more than 24 tablets (12 g), we admit.
Q.54. A 47 year old man was involved in a road traffic accident and was brought to the emergency department by ambulance. He has signs of respiratory distress and has abdominal and chest pain. On physical examination, no breath sounds are heard over the entire left lung field. A nasogastric tube is seen curled into the left chest on a chest X-ray. What is the most likely diagnosis?
Correct Answer : B
Diaphragmatic rupture is a tear of the diaphragm. It is usually secondary to blunt trauma. The usual case would be someonein a car passenger seat involved in a car accident where the seat belt compression causes a burst injury directed to the diaphragm. The pressure within the abdomen rises so quickly with a sudden blow to the abdomen causing a burst in the diaphragm. It is commonly on the left side.
Signs and symptoms include chest and abdominal pain and respiratory distress.
Breath sounds on the side of the rupture may be diminished. Bowel sounds may be heard in the chest. To your knowledge, it is important to remember that chest X-ray to diagnose diaphragmatic rupture is quite unreliable and has low sensitivity and low specificity.
However, there are specific signs detectable on X-ray which should raise suspicion. Diaphragmatic rupture is suspected in any patient with blunt trauma and a raised left hemi diaphragm on a chest X-ray.
Air fluid levels in the chest may also be seen. A nasogastric tube from the stomach may appear on the film in the chest cavity. This particular sign with the NG tube curled into the left chest is pathognomonic for diaphragmatic rupture, but it is rare. A thoracoabdominal CT scan is usually a diagnostic.
Q.55. A 49 year old man presents to the Emergency Department with a productive cough and fever. When asked about urinary symptoms, he says he has not passed much urine today. On examination, he appears drowsy. There are crackles heard on the right base of his lungs. His heart rate is 105 beats/minute, and respiratory rate is 25 breaths/minute. His blood results show the follow: Haemoglobin 130 g/L White cell count 15 x 109/L Platelets 300 x 109/L CRP 168 mg/L Serum urea 13 mmol/L Serum creatinine 240 micromol/L .What is the most appropriate management?
Correct Answer : A
This man should be treated for chest sepsis. The oliguria that he has is due to an acute kidney injury secondary to sepsis.
Always remember your sepsis 6. Intravenous fluid such as normal saline or Hartmann’s would be the most appropriate here.
Q.56. A 55 year old man tripped and fell onto a barbecue pit in the park and burnt his left arm. He is brought in by his son to the Accidents & Emergency 30 minutes later. Below is the picture of the patient’s arm. He is fully conscious. He denies alcohol intake during the barbecue party. Pain relief has been administered. What is the most appropriate management?

Correct Answer : D
The picture shows a red arm but there are no blisters seen so this can be classified as a superficial burn. Most superficial burns if dressing is required can be managed with a simple non-adherent dressing plus an absorbent layer (e.g. a dressing pad) on top and a bandage to secure the dressing.
He can then be managed as an outpatient with a twice-a-week wound inspection.
Other things to take note of are checking the tetanus status of the burnt victim and giving tetanus toxoids if required. This is because burns are considered tetanus-prone, especially in cases like this where it involves a barbeque pit in the park.
PARTIAL AND FULL-THICKNESS BURNS :
- Superficial (epidermal) burns are red and painful but do not blister.
- Partial thickness burn ranges from blistering to deep dermal burns.
- The appearance is shiny and the sensation is intact.
- Capillary refill blanches.
- Full-thickness burns have a leathery or waxy appearance. They may be white, brown, or black. There are no blisters and sensation is lost so they do not feel any pain. There is no capillary refill.
Q.57. Parents of an 11 month old baby want to know the steps for CPR for their child as they have recently attended a funeral of their neighbour’s child who died very suddenly after an episode of apnoea. They want to know what they can do if an event like this occurs. What is the most appropriate advice in regards to CPR for their child?
Correct Answer : B
Note: that if you were a paediatrician you should be doing a C: V ratio of 15:2 as you potentially need to resuscitate children as part of your role. Note there are two options for chest compression.
Either index and middle finger of one hand or grip the chest in both hands in such a way that two thumbs can press on the lower third of the sternum.
Q.58. A 13 year old boy presents with recurrent episodes of facial and tongue swelling. He also complains of abdominal pain which occurs occasionally. His father has had similar episodes. What is the most likely diagnosis?
Correct Answer : A
Hereditary angioedema (C1 esterase inhibitor deficiency) :
• Rare genetic condition causing episodes of angioedema which includes life-threatening laryngeal oedema.
The stem would usually include :
• A positive family history
• Onset from childhood of episodes of angioedema affecting the larynx. Sometimes impairing respiration and GI system, causing abdominal pain, and vomiting. Confirmed by:
• Low levels of C-esterase inhibitor and complement studies during the acute episode.
Q.59. A 23 year old girl presented with shortness of breath, perioral paraesthesia and carpopedal spasm 20 minutes after a huge argument with her boyfriend. She denies chest pain. What is the most appropriate next course of action?
Correct Answer : C
Hyperventilation is an abnormally increased pulmonary ventilation. Patients with hyperventilation syndrome may feel that they are not able to breathe sufficient air. In reality, their oxygenation in the arterial blood is within normal limits.
Due to the rapid breathing, carbon dioxide levels fall which results in respiratory alkalosis. The complaint is usually paroxysmal rather than continuous.
The patient may report breathlessness, followed by paraesthesia and/or tetany in fingers, distal limbs, and mouth. Some patients report chest pain.
Palpitations, wheezing, and sweating are also seen. This may be accompanied by a panic attack. If this becomes more severe, arterial hypocapnia may trigger cerebral vessel vasoconstriction leading to dizziness, faintness, headaches, and loss of consciousness.
After losing consciousness, breathing and blood gases return to normal which restores cerebral blood flow. The patient would shortly recover after that.
Management includes rebreathing into the paper bag to terminate the attack. Other methods include deliberately slowing down the breathing rate. Medications can also be used. Benzodiazepines can be used in acute situations if severe.
Q.60. A 23 year old male arrives at Accident & Emergency in an ambulance. The paramedics state that the patient was involved in a road traffic accident and that a group of bystanders called the ambulance. The patient was unconscious when the ambulance arrived. Upon physical examination, it is revealed that the patient’s abdomen has numerous bruises all over it and that he has cold, clammy and pale peripheries. His vitals are as follows: Blood pressure 70/40 mmHg Heart rate 130 beats per minute Temperature 36.4*C. What is the most likely cause of his cold and clammy peripheries?
Correct Answer : D
This question is testing basic physiology. Acute pain or trauma will activate your body’s sympathetic nervous system. This will cause many of the body’s blood vessels to constrict and will cause the heart to increase the rate and force of contraction.
This patient has just experienced a traumatic accident and is probably bleeding internally, as is evidenced by the abdominal bruising. Excessive blood loss results in a drop in blood pressure.
However, the single most likely cause of his cold and clammy peripheries is the activation of the sympathetic nervous system causing constriction of the blood vessels. This is due to a compensatory mechanism by the body to preserve itself.
Although severe trauma or stressful events can elevate cortisol levels, this is not a direct reason as to why this patient has cold and clammy peripheries.
Q.61. A 19 year old man is rushed into A&E by his friends who left him immediately before they could be interviewed by the medical staff. He is semiconscious. His respiratory rate is 7/min, blood pressure is 120/75 mmHg, and pulse rate is 60 bpm. He is noted to have needle track marks on his arms and he has pinpoint pupils. What is the most appropriate management?
Correct Answer : B
Coma with slow respiration and pinpoint pupils is typical of opioid poisoning, give naloxone. The other points that point towards an opioid overdose in this question are:
• Friends who did not want to be interviewed by medical staff
• Reduced consciousness
• Low respiratory rate
• Needle track marks on his arm.
Q.62. A 35 year old man had a fight in a bar which involved blunt trauma to his head. Since the injury, he has developed bleeding from the auditory meatus associated with ringing and hearing loss in his ear. Clear fluid is seen coming from his nose. What is the most appropriate investigations of choice?
Correct Answer : C
Any suspicion of a base of skull fracture should prompt a CT scan of the brain. The bleeding from the ear from the auditory meatus and rhinorrhoea are signs of basilar skull fracture.
SIGNS OF BASILAR SKULL FRACTURE :
Often a clinical diagnosis. One or more of the following may be seen:
• Bilateral orbital bruising confined to the orbital margin (‘Raccoon eyes’)
• Haemotympanum or bleeding from the auditory meatus
• Cerebrospinal fluid otorrhoea or rhinorrhoea
• Battle’s sign : Bruising over the mastoid process without local direct trauma.
Q.63. An 8 year old swallowed 12 tablets of paracetamol 4 hours ago. Serum paracetamol levels when tested are above the line on the nomogram. What is the next most appropriate action?
Correct Answer : D
The 4-hour paracetamol level is above the treatment line on the nomogram, so intravenous N-acetylcysteine should be given.
Paracetamol tablet = 500 mg, 12 tablets = 6 g of paracetamol. A significant overdose is > 75 mg/kg within 24 hours.
Management of paracetamol poisoning: N-acetylcysteine is the antidote for paracetamol overdose.
Oral activated charcoal 1 g/kg can be used up to 1-hour post-ingestion of paracetamol if the dose is > 150 mg/kg in 24 hours. It reduces serum levels by limiting gastric absorption.
This patient took the paracetamol overdose 4 hours ago and has a critical level of paracetamol, therefore activated charcoal is not appropriate here.
Blood to test on admission FBC, U&E, LFT, INR, venous blood gas, glucose, Serum paracetamol level at 4 hours post-ingestion.
N-acetylcysteine should also be started immediately.
Q.64. A 70 year old male presents with a 2 day history of productive cough and shortness of breath. He complains of chills and rigors. He is ill-looking. He has a temperature of 38.5°C, respiratory rate of 26 breaths/minute, and a pulse rate of 125 beats/min. His blood pressure is 88/45 mmHg and oxygen saturation is 90% on room air. On auscultation, bronchial breath sounds are heard in the periphery. He is given a fluid challenge of 1L normal saline. His blood pressure post fluid challenge is 90/40 mmHg. What is the best term to use in this condition?
Correct Answer : B
Although infection, sepsis, severe sepsis, and SIRS are the correct terms to use, the best term in this case is septic shock. Septic shock is defined as severe sepsis with persistently low blood pressure which has failed to respond to the administration of intravenous fluids.
Q.65. A 9 year old girl is brought by her mother to the A&E with stridor, wheezing and a rash. The young girl came from visiting her friend at her farm house. Her lips are beginning to swell. She has a very itchy generalized urticarial rash. What is the most appropriate treatment?
Correct Answer : C
Adrenaline is by far the most important drug in anaphylaxis and should be given as soon as possible. This girl is 9 years old hence 0.3ml of adrenaline is appropriate for her age (6 to 12 years old).
Once adrenaline is administered, hydrocortisone and chlorpheniramine should follow.
ANAPHYLAXIS ALGORITHM:
1. ABC
2. Give high-flow oxygen
3. Lay the patient flat
4. Adrenaline (epinephrine) intramuscularly in the anterolateral aspect of the middle third of the thigh.
ANAPHYLAXIS FEATURES :
The onset is usually in minutes or hours.
Respiratory:
• Swelling of lips, tongue, pharynx, and epiglottis. The fear here is that it may lead to complete upper airway occlusion
• Lower airway involvement is similar to acute severe asthma e.g. dyspnoea, wheeze (bronchospasm), chest tightness, hypoxia, and hypercapnia
Skin :
• Pruritus, erythema, urticaria, and angioedema
Cardiovascular :
• Peripheral vasodilation and increased permeability cause plasma leakage from the circulation, thus causing hypotension and shock.
Q.66. A 29 year old man is involved in a fight and receives a strong blow to his face. He is taken to the Accident & Emergency department where periorbital ecchymosis, enophthalmos and diplopia on upward gaze is noted. What is the most likely fractured bone?
Correct Answer : B
This is an orbital blowout fracture. The most common bone affected is the maxilla (orbital floor) followed by the ethmoid (medial wall). The diplopia on upward gaze is caused by impingement of the inferior rectus muscle.
Q.67. A 14 year old girl has been brought to the emergency department having taken an unknown amount of paracetamol 2 hours ago. She has no previous psychiatric history and is physically well. When would the most appropriate time to attain plasma paracetamol concentration levels following her presentation to A&E?
Correct Answer : A
If one takes more than an unknown amount of paracetamol, we admit Serum paracetamol concentration levels are attained at 4 hours post-ingestion if consumed > 150 mg/kg (or an unknown amount).
Q.68. A 5 year old boy was playing with his friends when one of his older friends carrying a kettle of hot water accidentally tripped and fell with the hot water spilling all over the child’s left arm. His parents quicky ran tepid tap water over his right arm for 10 minutes. He was then brought to the Paediatric A&E immediately by his parents. On assessment, partial thickness burns are seen with multiple blisters over his left arm, forearm and hand. What is the most appropriate management?
Correct Answer : D
This patient has a scald.
Scalds and burns are graded and managed in the same way. Analgesia should be the first thing to be prescribed and this patient should then be referred to a specialized burn service.
Most of them would include the following criteria:
• More than 3% of total body surface area partial thickness burn
• Burns involving the face, hands, feet, genitalia, perineum, or major joints
• All deep dermal and full-thickness burns
• All burns associated with electrical shock or chemical burns
• All burns associated with non-accidental injury
• All burns with inhalation injury
His hands are involved and there is a significant amount of body surface area involved, so it would be appropriate to refer to a specialist burn service which is run by experienced burn surgeons. Hand burns are not to be taken lightly because inadequate management can result in a serious disability.
Q.69. A 60 year old woman with history of a urinary tract infection, hypertension and gallstones presents to the emergency department. She complains of upper right abdominal pain, rigors and feeling unwell. Her urine dipstick is negative for white cell and nitrates. She has a temperature of 38.9°C. Her blood pressure is 88/55 mmHg, oxygen saturation of 92% on room air, pulse rate of 130 beats/minute and respiratory rate of 24 breaths/minute. What is the most likely diagnosis?
Correct Answer : C
This elderly woman has biliary sepsis probably due to a bile duct obstruction hence the upper right abdominal pain. As the urine dipstick is negative, it is unlikely that she has a UTI.
As she was never given IV fluids, we are unable to say that she has septic shock given that the term “septic shock” is defined as persistently low blood pressure which has failed to respond to the administration of intravenous fluids.
The very important term to note in this stem is “rigors”. Rigors are episodes of shaking or exaggerated shivering and are classically seen in 2 scenarios:
1. Bacteraemia such as seen in biliary sepsis or sepsis from pyelonephritis.
2. Malaria.
Q.70. A 7 year old girl has become acutely unwell while visiting a friend’s house and has been brought immediately to Emergency Department. Her friend’s mother gives a history of cooking a dessert with nuts. On assessment, she is fully conscious but has got stridor, wheeze and a sudden onset erythematous rash. Supplementary oxygen has been started. What is the immediate management?
Correct Answer : B
This is a classic case of anaphylaxis.
Administering 0.3 ml in 100U epinephrine intramuscularly would be the most immediate action as she is aged 6 to 12 years old.
From the stem, we can see that she has already been assessed which means the emergency team has already gone through the assessments for Airways, Breathing, and Circulation.
As she is fully conscious, her airway is likely patent and she does not need intubation.
Q.71. A 35 year old patient is brought to the emergency department after having a road traffic accident. He has bruises on his chest. A chest X-ray shows a widened mediastinum. He has a pulse of 129 beats/minute, a blood pressure of 80/40 mmHg and a respiratory rate of 34 breaths/minute. What is the most likely diagnosis?
Correct Answer : A
The condition is frequently fatal due to the profuse bleeding that results from the rupture. By far the most common site for tearing in traumatic aortic rupture is the proximal descending aorta.
The classical findings on a chest X-ray of an aortic rupture will be widened mediastinum and a displacement of the trachea. A widened mediastinum occurs when a traumatic pseudoaneurysm changes the contour of the mediastinum or more frequently haemorrhage or haematoma occurs.
Q.72. A 24 year old man was under the custody of police when he was punched. He is now cyanosed and unresponsive. What is the most appropriate action?
Correct Answer : C
In this question, the examiners want you to know the basics of life-threatening emergencies:
ABC – airway, breathing, and circulation should always be addressed first.
Q.73. A 19 year old boy presents with nervousness, a runny nose, watering eyes, tremors and muscle cramps. He says that he has also been having trouble sleeping and feels like he lacks mental clarity. He is a known drug addict and he claims that he has been trying to stop taking drugs for the past three days. He looks agitated and sweaty. What is the most likely drug that he has been taking?
Correct Answer : D
This patient’s signs and symptoms fit in with heroin withdrawal. Early signs include a flu-like syndrome like muscle cramps, watering eyes, and a runny nose.
Other symptoms like agitation, sweating, and insomnia are also part of heroin withdrawal. Some may argue that symptoms of cocaine withdrawal may be similar and the runny nose in this stem may be from the persistent post-nasal drip indicating that the user snorts cocaine as their preferred method of ingestion.
This is also a valid explanation but we feel that heroin is still a more appropriate answer.
Q.74. A 78 year old woman presents to the Emergency Department with severe epigastric pain and vomiting for the past 2 days. The pain is referred to her right shoulder. Generalized rigidity of the abdomen is noted when examining. She denies chest pain. She has a temperature of 37.2°C, a respiratory rate of 22 breaths/minute and a pulse of 102 beats/minute. Her medical history is significant for rheumatoid arthritis. What is the most appropriate initial investigation?
Correct Answer : A
The history of rheumatoid arthritis tells you that she is more than likely to be taking NSAIDs to manage the pain. This is a risk factor for peptic ulcer which if perforated gives the signs and symptoms that she is describing.
Perforation of a peptic ulcer causes an acute abdomen with epigastric pain that may progress to generalized rigidity. Shoulder tip pain suggest is seen in perforation. A diagnosis is made by taking an erect abdominal/chest X-ray.
Air under the diaphragm gives the diagnosis of a perforation.
Q.75. A 25 year old male with a history of frequent binge drinking presents 4 hours after having had a takeaway meal following a night’s heavy drinking. He complains of nausea and has vomited on several occasions. After the last vomiting episode, he vomited approximately a cupful of blood. On admission, he smells of alcohol, pulse of 100 beats/minute, blood pressure of 140/80 mmHg. He has some tenderness in the epigastrium. What is the most likely diagnosis?
Correct Answer : C
Please see Q-33
Q.76. A fit and active 60 year old man comes with an acute onset swelling of his calf. On examination, the mid calf measures 3 cm larger in girth than the other calf. There is no redness and the skin appears well perfused. Peripheral pulses are present. He describes worsening pain when the ankle is dorsiflexed. What is the most likely diagnosis?
Correct Answer : D
The most likely diagnosis in this case is likely to be a DVT.
A calf swelling with a positive Homan’s sign (pain when the ankle is dorsiflexed) is commonly seen in deep vein thrombosis (DVT) and popliteal cyst ruptures. Although there is no risk in the stem for a DVT, it is still more common than a popliteal cyst rupture. Popliteal cyst rupture can also cause calf pain and enlargement which is similar to a DVT.
However, popliteal cyst rupture would initially start with swelling and discomfort behind the knee (representing the popliteal cyst) which is then followed by rupture which results in pain and swelling in the calf that mimics DVT. Since there is no evidence of swelling and discomfort behind the knee, pick the more common condition of the two, which would be a DVT.
Imaging such as an ultrasound would differentiate between the two. Baker’s cyst (popliteal cyst) would present as a swelling behind the knee rather than a calf swelling and is generally asymptomatic though it may cause discomfort.
Achilles Tendon rupture would present with the patient giving a history of a ‘popping’ sound at the time of rupture with pain around the ankles and decreased plantar flexion.
Q.77. A 32 year old man is brought to A&E by his wife with symptoms of chest pain, abdominal pain, palpitations, nausea and altered mental status. His wife says that she found a bottle of empty medication on the toilet floor and it is likely that he consumed the whole bottle. He has a respiratory rate of 34 breaths/minute. His arterial blood gas shows the following: pH 7.21 pCO2 3.0 kPa PO2 13 Bicarbonate 18 mmol/L. What is the most likely medication that he could have taken?
Correct Answer : B
The clinical picture here represents metabolic acidosis. This is likely from aspirin poisoning. With aspirin overdose, they initially hyperventilate leading to respiratory alkalosis.
The initial phase could last up to 12 hours. Following that, progressive metabolic acidosis occurs. This usually occurs around the 24-hour mark after ingestion for adults.
Paracetamol overdose may also cause metabolic acidosis in the early and late course of poisoning and thus paracetamol as an answer is not completely wrong.
However, aspirin overdose is a more recognized cause of metabolic acidosis compared to paracetamol.
Overdose of Enalapril (ACE inhibitor) or Diclofenac (NSAIDs) causes metabolic alkalosis and not acidosis.
Overdose of Benzodiazepines would cause respiratory acidosis. If suspecting metabolic acidosis do a blood gas.
Q.78. An 18 year old girl has taken an unknown dose of paracetamol yesterday. She cannot remember the exact time she took the tablets. She is extremely anxious but otherwise asymptomatic. She has no relevant past medical history. She weighs 55 kg. Her blood pressure is 120/75 mmHg, heart rate is 105 beats/minute and SaO2 is 99% on air. Which is the most appropriate next step?
Correct Answer : C
N-acetylcysteine infusion should be started if there is doubt over the time of paracetamol ingestion, regardless of the plasma paracetamol concentration.
Activated charcoal should be considered if the overdose has been taken if the overdose has been taken within 1 hour.
Gastric lavage would always be the wrong answer when dealing with paracetamol poisoning.
Discharge home only if ingestion of paracetamol is < 150 mg/kg in a patient with no hepatic risk factors.
Q.79. A 16 year old female teenager was brought to the emergency department after being stabbed on the upper right side of his back 2 hours ago. An erect Chest X-ray revealed homogeneous opacity on the lower right lung. The trachea is centrally placed. She has a blood pressure of 80/60 mmHg, a pulse of 122 beats/minute, and a respiratory rate of 34 breaths/minute. What is the most likely diagnosis?
Correct Answer : D
Haemothorax:
• Blood accumulates in the pleural cavity
Clinical features:
Similar to that seen in traumatic pneumothorax, except for the following:
• Dullness to percussion over the affected lung
• Signs and symptoms of hypovolaemia if massive haemothorax
Investigations:
• Chest X-ray shows an increased shadowing on a supine X-ray, with no visible fluid level
Treatment:
• Oxygen
• Insert 2 large venous cannulas and send blood for cross-matching
• Evacuation of blood may be necessary to prevent the development of empyema; thus, a chest tube is needed and is often placed low.
Usually, the lung will expand and the bleeding will stop after a chest tube is inserted.
• Surgery to stop the bleeding is seldom required. The lung is the usual source of bleeding.
• Since it is a low-pressure system, the bleeding usually stops by itself.
Q.80. A 33 year old man was admitted to the emergency department after a head injury. On arrival, he has a Glasgow Coma Scale score of 15. A few hours later, his Glasgow Coma Scale score drops to 12. What is the most appropriate immediate action?
Correct Answer : C
A CT head would be appropriate. The initial GCS of 15 followed by a later GCS of 12 are suggestive of intracranial haemorrhage.
Q.81. A butcher comes to the emergency department after accidentally stabbing his groin with a knife. He tried to use a towel to stop the bleeding but has bled so much that the towel is now soaked with blood. His blood pressure is 80/50 mmHg and pulse is 130 beats/minute. What percentage of circulatory blood did he lose?
Correct Answer : D
Please see Q-29
Q.82. A 4 year old boy is brought into the emergency department by his mother. He has reportedly been drowsy and pyrexial for the past 2 hours. Whilst waiting to be seen he becomes unresponsive. He makes no respiratory effort. 5 rescue breaths were given. A brachial or carotid pulse cannot be detected. What is the most appropriate ratio of compressions to ventilations as part of cardiopulmonary resuscitation for this patient?
Correct Answer : D
15 compressions to 2 breaths with the nose pinched is the answer.
For a child over 1-year-old, pinch the soft part of his nose closed with the index finger and thumb of your hand on his forehead when giving rescue breaths.
Q.83. A 22 year old man was rushed into the emergency department. He describes recurrent episodes of fearfulness, palpitations, with peri-oral tingling and cramping of the hands. His symptoms last 5-10 minutes. He is worried he may be having a heart attack. An ECG shows sinus tachycardia. He has a respiratory rate of 34 breaths/minute. What is the most appropriate immediate intervention?
Correct Answer : A
Please see Q-59
Q.84. A middle aged man is seen having stridor and wheezing a few minutes after consuming a cake at a restaurant. The paramedics arrive at scene and suspect anaphylaxis. What is the most appropriate route of administration of adrenaline in this case?
Correct Answer : A
In case of anaphylaxis, adrenaline is given intramuscularly.
Q.85. A butcher comes into A&E after accidentally stabbing his groin with a knife. He tried to use a towel to stop the bleeding but has bled so much that the towel is now soaked with blood. His blood pressure is 75/40 mmHg and a pulse is 120 beats/minute. What is the most appropriate initial management?
Correct Answer : D
In reality, we would start IV fluids as soon as possible and get a crossmatch for a blood transfusion. Thus, many argue that option A blood transfusion would be correct. Although debatable, the question here does ask for the most appropriate initial management.
IV fluids would be more appropriate given that he is hypotensive and IV fluids would be more appropriate given that he is hypotensive and IV normal saline is readily available in the A&E departments. Packed RBC takes time as staff would need to call the blood transfusion lab and order blood.
Q.86. A 35 year old woman was admitted to the medical ward for pneumonia and was started on intravenous co-amoxiclav. 10 minutes after starting co-amoxiclav, she develops a generalised itchy rash and becomes breathless. On auscultation, a widespread wheeze is heard throughout her lung fields. Her heart rate is 138 beats/minute and her blood pressure is 70/55 mmHg. What is the most appropriate management?
Correct Answer : B
This is a case of anaphylaxis.
Treatment should involve discontinuing the antibiotic, giving 100% oxygen, and maintaining the airway. Since there is evidence of shock and respiratory compromise, intramuscular epinephrine (adrenaline) should be administered.
Intravenous antihistamines such as chlorpheniramine 10 mg and hydrocortisone 100-200 mg are commonly prescribed as well as they reduce the severity and duration of symptoms.
If this patient, does not have an acute drop in his blood pressure and does not have airway swelling or evidence of respiratory difficulty, intravenous chlorpheniramine and hydrocortisone would suffice and we could withhold the adrenaline.
Q.87. A 33 year old male was involved in a road traffic accident. He was sitting in the front passenger seat and wearing a seatbelt when the car was hit on the left side. On examination there are bruises on the left side of his chest and abdomen and there is marked abdominal distension. His pulse is 130 beats per minute and his blood pressure is 60/40 mmHg. What is the most likely injury?
Correct Answer : C
The person has suffered a blunt trauma on the left side of his chest and abdomen. When there is a history of road traffic accidents, the presence of bruises (especially over the left chest wall or left upper quadrant), abdominal distention, and cardiovascular compromise,
it is important to consider splenic injury. A splenic injury which includes a ruptured spleen is any injury to the spleen and it occurs commonly after a road traffic accident.
Other features to note in splenic rupture would be the falling blood pressure and the rising pulse. Abdominal tenderness and diminished bowel sounds are also seen.
Plain abdominal X-ray would show a loss of left psoas shadow and this is confirmed by a CT scan.
FAST scans in the emergency department setting have the potential to reveal free peritoneal fluid but while this is helpful, it should not delay CT imaging or surgical intervention.
Q.88. A 10 year old is brought in by ambulance after having been rescued from a house fire. There is soot in his mouth and he has difficulty breathing. What is the most appropriate management?
Correct Answer : B
Please see Q- 46
Q.89. A 68 year old woman has had a carotid angiogram done and it was subsequently decided to place two stents in her carotid arteries. The access for the carotid endarterectomy procedure was obtained via the femoral arteries. Post-operatively, the patient was found to have bilateral groin swelling. The patient also appeared weak and anxious and her blood pressure was measured at 85/55 mmHg. What is the most appropriate immediate step in management of this patient?
Correct Answer : D
The question is testing your knowledge of your ABCs and nothing else. Pay attention to the way that the exam gives you a lot of information in the stem; this is designed to ‘waste your time’.
Many people end up failing the exam because of poor time management. This question has nothing to do with the fact that she underwent carotid stenting. All you need to pinpoint for this question is that this post-operative patient is going into shock.
The next most appropriate step would be to apply pressure on the point of swelling as it is most likely that there is a bleed beneath the swelling.
Q.90. A 33 year old man was involved in a road traffic accident. He has acute abdominal pain. On abdominal examination, there is bruising along the site of the portion of the seatbelt. His abdomen is very tender. A computerised tomography scan of his abdomen shows a subcapsular splenic haematoma. He has a pulse rate of 90 beats/minute, respiratory rate of 24 breaths/minute, and a blood pressure of 120/80 mmHg. What is the most appropriate action?
Correct Answer : D
A subcapsular splenic haematoma as diagnosed by the preoperative CT scan is neither a predictor for delayed splenic rupture, nor by itself an indication for operative management of the injured spleen in a haemodynamically stable patient.
Observation would be the most appropriate action.
Focused abdominal sonography (ultrasound) for trauma (FAST) is regarded as the investigation of choice for early diagnostic investigations in patients with suspected blunt abdominal trauma and it is regarded as the investigation of choice for unstable patients.
Free fluid in a haemodynamically unstable patient indicates the need for emergency laparotomy.
However, this patient is stable and does not warrant an immediate laparotomy.
The FAST scans are typically used to enhance the speed of trauma assessment, reduce the number of CT scans, and cut costs.
However, this patient has already had a CT scan, thus an ultrasound scan would not give any added information concerning an acute abdomen.
Solid organ injury in haemodynamically stable patients can often be managed without surgery but would require observation for a while before discharge.
Q.91. A 35 year old man with a known peanut allergy presents to Accident & Emergency after having unknowingly ingested a cupcake with nuts. He has widespread wheezes bilaterally on auscultation and he is experiencing stridor and dyspnoea. His lips and tongue have also started to swell. Intravenous access has been established. His heart rate is 110 beats/minute and his blood pressure is 88/59 mmHg. What is the most appropriate immediate treatment for him?
Correct Answer : A
Even though IV access has been established, the best treatment to resolve his hypersensitivity reaction is STILL intramuscular adrenaline.
Q.92. A 45 year old man with a medical background of epilepsy is brought in by ambulance as he has been having an ongoing generalized tonic clonic seizure for the past 40 minutes. He is unconscious, has noisy breathing and looks cyanosed. He has intravenous access. What is the most appropriate initial management?
Correct Answer : C
A continuous generalized seizure lasting for more than 30 minutes has a high risk of cerebral damage. The first part of management is always to establish a clear airway.
Q.93. A 33 year old lady who is an opiate drug addict wants to quit her drug abuse problem. She is supported by her friends and family. What is the most appropriate treatment to combat withdrawal symptoms?
Correct Answer : A
Methadone is used to help combat withdrawal effects for opiate misusers. Methadone is an opioid medication and
it reduces withdrawal symptoms in people addicted to heroin. It does not cause the “high” associated with the drug addiction and thus it is especially helpful for opiate addicts to help them quit the drug addiction problem.
Q.94. A 17 year old girl has taken an overdose of 30 paracetamol tablets 3 hours ago. She is extremely anxious but otherwise asymptomatic. She has no past medical history. She weighs 50 kg. BP 120/70 mmHg, heart rate is 100 bpm, SaO2 98% on air. Which is the most appropriate next step?
Correct Answer : D
Blood taken before 4 hours is unreliable because the drug is still being absorbed and distributed. The N-acetylcysteine infusion does not need to be started unless the time of the overdose is unknown or has been staggered over a few hours. Initial symptoms are often limited to nausea and vomiting.
Activated charcoal should be considered if the overdose has been taken if the overdose has been taken within 1 hour.
Gastric lavage would always be the wrong answer when dealing with paracetamol poisoning.
N-acetylcysteine infusion should not be started until the blood test has been taken at 4 hours and results are above a single treatment line.
Q.95. A 36 year old man rescued from a building on fire presents with headache, vertigo, nausea, confusion and vomiting. He complains of feeling weak. He is without any evidence of burns or external injury. What is the most appropriate management?
Correct Answer : B
As this patient is conscious, we just need a tight-fitting mask with 100% oxygen.
Q.96. A 15 year old girl is admitted in the medical ward after taking 28 tablets of paracetamol with a large amount of alcohol. Her plasma paracetamol concentration taken 4 hours post ingestin is just below the concentration that would suggest treatment with N-acetylcystein. What is the most appropriate next course of action?
Correct Answer : C
Referral to a psychiatric team would be the most accurate option here. Acute alcohol consumption is an inhibitor of the P-450 enzyme system. Since she has consumed a large amount of alcohol acutely, the risk of fatal effects of paracetamol poisoning would be reduced.
Not to mention that her plasma paracetamol concentration is below the treatment line thus she does not need to start N-acetylcysteine. She does however need a psychiatric evaluation before she leaves the hospital as she was trying to harm herself.
Q.97. An 8 year old boy is rushed into A&E cough in, cyanosed and with a urticaria rash. His mother tells the staff that he began to cough after eating a cookie at a garden party. What is the most likely diagnosis?
Correct Answer : A
This boy is having anaphylaxis which is a type of allergic reaction.
The other options are far less likely: Aspiration of food. Although this makes sense and many will choose this, aspiration for food will not account for the urticarial rash seen on this boy.
Diffuse Esophageal Spasms usually presents with intermittent chest pain and dysphagia.
The pain can simulate that of a myocardial infarction but bears no relationship with exertion. There is no relationship with eating, ruling out odynophagia. The pain can be precipitated by drinking cold liquids.
Achalasia presents with slowly progressive dysphagia. Initially worse for liquids than solids.
Q.98. A 65 year old man who has been prescribed amitriptyline for depression, has been brought to the emergency department with suspected overdose. He is not giving a coherent history. His skin looks dry and flushed. On examination, his pupils are dilated. His blood pressure is 90/60 mmHg. What is the most appropriate immediate action?
Correct Answer : C
ECG monitoring for TCA overdose is essential.
The important ECG finding suggestive of TCA poisoning is QRS widening. Broad complex tachycardias may occur which are life-threatening.
Q.99. A 12 year old comes into the emergency department with severe burns all over his body from a house fire. There is oropharyngeal swelling and soot in the mouth. He is in severe pain. What is the most appropriate management?
Correct Answer : B
Please see Q-46
Q.100. A 34-year-old alcoholic is found passed out in front of a local pub. The ambulance crew informs you that he was sweating when they found him and there were cans of cider lying empty around him. What is the initial stage of investigation?
Correct Answer : A
Alcohol-induced hypoglycemia can present in this case.
Q.101. A 33-year-old drug addict wants to quit. She says she is ready to stop the drug abuse. She is supported by her friends and family. What drug treatment would you give her?
Correct Answer : D
Methadone is a synthetic opiate primarily used in the detoxification and maintenance of patients who are dependent on opiates—particularly heroin.
Q.102. A young college student is found in his dorm unconscious. He has tachyarrhythmia and high fever. He also seems to be bleeding from his nose, which on examination shows a perforated nasal septum. What is the most likely diagnosis?
Correct Answer : B
Cocaine overdose.
Tachyrhythmia, High fever, and perforated nasal septum are suggestive features of cocaine overdose. Other features include dilated pupils, chest pain, anxiety, delirium, paranoia, etc. Cocaine can even lead to death, which can occur due to a multitude of complications, including heart attack, seizure, and stroke.
Q.103. An 18-year-old man is rushed into the ER by his friends who left him immediately before they could be interviewed by staff. He is semiconscious, RR (respiratory rate) 8/min, BP 120/70 mmHg, pulse 60 bpm. He is noted to have needle track marks on his arms and his pupils are small. What is the best initial treatment?
Correct Answer : B
This is opiate overdose. Due to their effect on the part of the brain that regulates breathing, opioids in high doses can cause respiratory depression and death.
An opioid overdose can be identified by a combination of three signs and symptoms referred to as the “opioid overdose triad”. The symptoms of the triad are pinpoint pupils, unconsciousness, and respiratory depression.
Combining opioids with alcohol and sedative medication increases the risk of respiratory depression and death, and combinations of opioids, alcohol, and sedatives are often present in fatal drug overdoses.
Q.104. An 18-year-old female just received her A-Level results and she didn’t get into the university of her choice. She was brought into the emergency department after ingestion of 24 paracetamol tablets. Examination: confused and tired. Initial management has been done. Investigations after 24 hours: normal CBC, ABG pH7.1, PT 17s, Bilirubin 4 umol/L, creatinine 83 umol/L. What is the next step in management?
Correct Answer : D
Criteria for liver transplantation in paracetamol-induced acute liver failure.
Arterial pH <7.3 or arterial lactate >3.0 mmol/L after adequate fluid resuscitation, OR
If all three of the following occur in a 24-hour period: Creatinine >300 mol/L, PT >100 seconds (INR >6.5), Grade III/IV encephalopathy.
Q.105. Patient with history of alcoholism, ataxic gait, hallucinations and loss of memory. He is given acamprosate. What other drug can you give with this?
Correct Answer : B
These are features of wernicke’s encephalopathy.
Q.106. A 16-year-old girl is admitted after taking a paracetamol over dose 4 hour ago. She has consumed large amounts of alcohol. Her plasma paracetamol concentration is just below the concentration that would suggest treatment. What should be the treatment option for her?
Correct Answer : A
Short-term acute alcohol consumption causes enzyme inhibition as in the present case and even then paracetamol level is under treatment level. So the patient's drug level is on the safe side but simultaneous drug overdose and alcohol consumption needs psychiatric evaluation and hence the option here is A.
Q.107. In a man who is neglected and alcohol dependent, which high suicidal risk, which factor can increase this risk further?
Correct Answer : B
He is already alcohol dependent.
SSRI has a well-known risk of developing suicidal ideation. Arrest and court involvement is also a risk factor and in a neglected person it is also a strong risk factor.
Q.108. A young woman was brought to the hospital. On examination she has low temperature and tremor. She says when she closes her eyes, she can see colors. What drug has been used?
Correct Answer : B
LSD
Q.109. A 40-year-old man who is a chronic alcoholic with established liver damage, has been brought to the hospital after an episode of heavy drinking. He is not able to walk straight and is complaining of double vision and is shouting obscenities and expletives. What is the most likely diagnosis?
Correct Answer : C
Wernicke’s encephalopathy. Triad of i) ophthalmoplegia ii) ataxia iii) confusion.
Q.110. A 50-year-old smoker and heavy drinker presents with complaints of racing heart. A 24 hour ECG comes out normal. What is your next step in management?
Correct Answer : B
Smoking and alcohol excess can cause palpitation without any recognizable arrhythmia and for this, no treatment is required.
Q.111. A teenage girl who was ‘fine’ until her boyfriend said he didn’t want the relationship anymore. She took 10 tablets of paracetamol in front of his mother after taking alcohol. What should you do?
Correct Answer : A
10 tablets of paracetamol is not a life-threatening toxic dose and simultaneous drug overdose and alcohol consumption need psychiatric evaluation.
Q.112. A 65-year-old known alcoholic is brought into hospital with confusion, aggressiveness and ophthalmoplegia. He is treated with diazepoxide. What other drug would you like to prescribe?
Correct Answer : D
Vit B complex
Q.113. A 21-year-old girl looking unkempt, agitated, malnourished and nervous came to the hospital asking for painkillers for her abdominal pain. She is sweating, shivering and complains of joint pain. What can be the substance misuse here?
Correct Answer : B
Agitation, nervousness, abdominal cramps, sweating, shivering and piloerection, and arthralgia these are features of heroin withdrawal.
Q.114. A 54-year-old man with alcohol dependence has tremor and sweating 3 days into a hospital admission for a fracture femur. He is apprehensive and fearful. What is the most appropriate treatment?
Correct Answer : B
Chlordiazepoxide.
The Drug of choice for withdrawal is chlordiazepoxide because of its long action.
The dose depends on the grade of severity of withdrawal, which is calculated on the basis of the CIWA-Ar score (Not required, just for additional information). A short?acting benzodiazepine such as oxazepam or lorazepam may be used in individuals with impaired liver function.
Q.115. A 30-year-old man complains of episodes of hearing music and sometimes threatening voices within a couple of hours of heavy drinking. What is the most likely diagnosis?
Correct Answer : D
Alcohol hallucinosis can occur during acute intoxication or withdrawal. It involves auditory and visual hallucinations, most commonly accusatory or threatening voices.
Q.116. A 72-year-old woman who had a repair of strangulated femoral hernia 2 days ago becomes noisy, aggressive and confused. She is afebrile, CBC normal apart from raised MCV. What is the most likely diagnosis?
Correct Answer : B
Electrolyte imbalance may cause confusion but not aggressiveness; infection toxicity will cause high fever, low BP, rash, etc which are absent here.
Abstinance from alcohol in the hospital caused delirium tremens (chronic alcoholism is supported by high MCV) here.
Q.117. An 18-year-old previously well student is in his 1st year at university. He has been brought to the emergency department in an agitated, deluded and disoriented state.What is the most probable reason for his condition?
Correct Answer : A
Young age and 1st year in university is likely to point towards drug abuse.
Q.118. A 62-year-old man who was admitted for surgery 3 days ago suddenly becomes confused. His attention span is reduced. He is restless and physically aggressive and picks at his bed sheets. What aspect of the patients history recovered in his notes is most likely to aid in making the diagnosis?
Correct Answer : A
Abstinence from alcohol in the hospital leads to delirium tremens.
Q.119. A 20-year-old boy is brought by his parents suspecting that he has taken some drug. He is agitated, irritated and can’t sleep. Exam: perforated nasal septum. Which of the following is the most likely to be responsible for his symptoms?
Correct Answer : B
Drug abuse with perforated nasal septum indicates cocaine abuse.
Q.120. A 34-year-old man with multiple sclerosis has taken an over dose of 100 tablets of paracetamol with intent to end his life. He has been brought to the emergency department for treatment but is refusing all intervention. What is the next step?
Correct Answer : B
Evaluate patient's capacity to refuse treatment.
Q.121. A 37-year-old woman was admitted for femur fracture repair after a RTA. On the 4th post-operative day she became confused and starts picking on her bed sheets and complains of seeing spiders all over. What is the most likely diagnosis?
Correct Answer : A
Withdrawal of alcohol due to hospital admission leads to delirium tremens.
Wernicke’s encephalopathy has a triad of :
i) mental confusion
ii) abnormal eye movements &
iii) unsteady gait.
Korsakoff's syndrome cannot be diagnosed until the person has stopped drinking alcohol for several weeks, to enable the immediate symptoms of alcohol intoxication and withdrawal to subside.
Features of Korsakoffs psychosis :
i) dementia, loss of short-term memory
ii) difficulty in acquiring new information or learning new skills
iii) personality change
iv) confabulation
Q.122. A 50-year-old man complains of being pursued by the police for a crime he denies committing. He has poor concentration and impaired short-term memory. He admits to drinking large amounts of alcohol for the last 20yrs. What is the most probable diagnosis?
Correct Answer : D
Dementia, i.e. short term memory loss is seen in korsakoff psychosis.
Q.123. An alcoholic who has completely given up drinking starts hears voices. What is the most appropriate treatment?
Correct Answer : B
A case of delirium tremens. Treated with benzodiazepines like chlordiazepoxide or diazepam.
Q.124. A 62-year-old male is brought to the emergency department by his daughter because of his persistent lying. He is a known alcoholic and has been admitted recently with delirium tremens. On questioning, he denies any problem with memory. He knows his name and address and states that he was at the betting shop this morning, but his daughter interjects calling him a liar explaining that he was at her home. What is the most likely diagnosis?
Correct Answer : D
Korsakoff's psychosis.
In Korsakoff psychosis there is confabulation (a memory disturbance is defined as the production of fabricated, distorted, or misinterpreted memories about oneself or the world, without the conscious intention to deceive) which may present like this.
Q.125. A 65-year-old known alcoholic is brought into the hospital with confusion, aggressiveness and ophthalmoplegia. He is treated with diazepoxide. What other drug would you like to prescribe?
Correct Answer : D
This is a case of Wernicke’s encephalopathy which is treated with Vit B complex.
Q.126. A 53-year-old man was admitted to the hospital for investigation of hemoptysis. 3 days after admission he developed alternating state of consciousness, ataxic gait and some visual problems. What is the most appropriate management of this patient?
Correct Answer : D
This is a case of Wernicke’s encephalopathy (as hospital admission prevented him from consuming alcohol) and the treatment for this is high potent vitamins.
Q.127. A young lady after a heavy bout of drinking last night comes to the emergency department with dizziness, abdominal pain, vomiting blood with cool peripheries. After initial resuscitation, oxygen and fluids, she still continues to bleed with pulse 130 bpm and BP 85/58 mmHg. What would be your next best management?
Correct Answer : D
In young it is unlikely to get oesophageal varices rather than the bleeding is from probable gastric erosion and an endoscopy should be done diagnostic and therapeutic- like sclerotherapy.
Q.128. A 25-year-old male with a history of frequent binge drinking presents 4 hour after having had a take away meal following a nights heavy drinking. He complains of nausea and has vomited on several occasions.After the last vomiting episode, he vomited approximately a cupful of blood. On admission, he smells of alcohol, pulse 100 bpm, BP 140/80 mmHg. He has some tenderness in the epigastrium. What is your diagnosis?
Correct Answer : B
Repeated retching and vomiting is a recognized cause of Mallory-Weiss tear which is supported here by vomiting a cupful of blood.
Q.129. An 8-year-old child swallowed 12 tablets of paracetamol 4 hour ago. Serum paracetamol levels when tested were at critical level. What would you do next?
Correct Answer : B
As 4 hour paracetamol level is at critical level start IV N-acety cystiene at once.
Q.130. A 24-year-old man has been found unconscious in an alleyway with a respiratory rate 6 bpm and heart rate 60 bpm. His pupils are constricted. What is the best treatment?
Correct Answer : B
Respiratory depression and miosis points towards opiates poisoning for which antidote naloxone is given.
Q.131. A 45-year-old man has been admitted for an elective hernia surgery. 3 days later he presents with agitation, sweating, aggressiveness, and complains of seeing snakes on the hosp wall. Chlordiazepoxide has been started for this patient. What is the most appropriate next step?
Correct Answer : D
Diagnosis is delirium tremens. So the following chlordiazepoxide thiamine is the appropriate next step.
Q.132. A lady who is alcohol dependent wants to quit but wants someone to encourage her. What would you do?
Correct Answer : B
Refer to social services
Q.133. A couple has just finished their detox regime and wants a drug with a pharmacological action to serve as a deterrent when they take alcohol. What drug is the appropriate choice?
Correct Answer : A
Disulfirum is a deterrent that makes alcohol intake unpleasant.
Q.134. A 33-year-old lady who is a drug addict wants to quit. She says she is ready to stop the drug abuse. She is supported by her friends and family. What drug treatment would you give her?
Correct Answer : D
Methadone is an opioid medication.
Methadone reduces withdrawal symptoms in people addicted to heroin or other narcotic drugs without causing the "high" associated with the drug addiction and thus used to help a drug addict to quit from drug addiction.
Q.135. A 78-year-old retired teacher was admitted for a hernioplasty procedure. After the operation he became agitated, aggressive and confused. What is the most appropriate management?
Correct Answer : B
Diagnosis is delirium tremens. Treatment is with chlordiazepoxide.
Q.136. A 46-year-old man who is a heavy drinker is brought to the emergency department in a drowsy state. He is responding vaguely to questions. Examination: nystagmus and hyperreflexia. MCV 103 fl. What is the most likely cause for his cognitive impairment?
Correct Answer : A
Presence of confusion and nystagmus indicates Wernicke’s encephalopathy caused by thiamin (vit. B1) deficiency.
Q.137. A 56-year-old alcoholic man who has increased the amount of alcohol he is using wants to attend his daughter’s wedding that is in 2 weeks. He is now coming to you for help. How would you help him?
Correct Answer : A
If the patient stops alcohol without any supportive treatment there will occur withdrawal symptoms.
For the presented situation, Acamprosate can help by stopping alcohol without producing withdrawal symptoms by restoring brain chemical derangement caused by alcohol which is responsible for withdrawal symptoms.
Q.138. A patient, 50-year-old smoker and heavy drinker, presents with complaints of racing heart. A 24 hours EKG comes out normal. What is your next step in management?
Correct Answer : B
Racing heart or palpitation is a common phenomenon in alcoholics which is not serious or harmful. So reassure the patient.
Q.139. A 77-year-old publican was admitted for an appendectomy. Post-operation he becomes confused, agitated and starts to pick at things. He is then given an IV drug which settles this confusion. Which of the following drugs was given for his confusion?
Correct Answer : B
Patient is alcoholic.
On admission abstinence from alcohol caused this withdrawal symptom. Agitation, confusion, and picking at things are suggestive of delirium tremens which is treated with Chlordiazepoxide.
Q.140. A 28-year-old woman with history of drug addiction wants to start a family and have a baby. She would like to stop taking heroin and asked for something to help her stay away from it. What drug treatment would you give her?
Correct Answer : C
Methadone is a powerful synthetic analgesic drug which is similar to morphine in its effects but less sedative and is used as a substitute drug in the treatment of morphine and heroin addiction.
Q.141. A homeless person is found wandering on the street. He had ataxic gait, nystagmus and opthalmoplegia. He looked unkempt and his clothes had a sweaty odour. He had a dry mucous embrane with a BP 118/70mmHg and PR (pulse rate) 90 bpm. Blood sugar level 8. Alcohol breath test -ve. What would the most important initial management?
Correct Answer : B
This is wernicke’s encephalopathy due to alcohol withdrawal, so Vit B complex should be given. This occurs mainly due to thiamine deficiency.
Q.142. A 45-year-old man has undergone detox and now wants a drug to stop him from craving alcohol. What medication would be that drug of choice?
Correct Answer : B
Disulfirum is a deterrent which does not reduce craving. Acramposate by restoring deranged brain chemical reduces craving.
Q.143. A 45-year-old man, known to be chronically addicted to alcohol, presents in the emergency department and reports two episodes of vomiting fresh bright red blood in the previous 6 hours. He estimated the volume blood vomited at each bout to be more than 500mls. Clinical exam: the radial pulse 120/min, BP 90/60 mmHg. There is no mass or tenderness in the epigastrium. The liver is palpable for 3 cm below the costal margin and not tender. The patient is not jaundiced. The physician resuscitates the patient with oxygen by face mask, rapid infusion of intravenous normal saline while he requests for haemoglobin level and whole blood for transfusion. Which is next appropriate step in management?
Correct Answer : D
The likely diagnosis is bleeding oesophageal varices which should be diagnosed by endoscopy and if needed stappling can be done with endoscope.
Q.144. A-16-years old boy was brought to hospital in a comatose state having taken methadone belonging to his sister. He was given naloxone and rapidly became alert. Some hours later, he gradually becomes semi-conscious again. What is the most likely reason for this patient becoming semi-conscious again in hospital?
Correct Answer : B
Methadone is eliminated from the body more slowly than naloxone.
Q.145. A 38-year-old man with longstanding alcohol dependence has vertigo and a tremor every morning. What is the most likely diagnosis?
Correct Answer : C
chronic alcohol abuse that leads to temporary or permanent cerebellar damage.
Q.146. A 22-year-old says she has taken about 40 tablets of paracetamol 3 hours ago. Her heart rate is 110 bpm, BP 110/80 mmHg and respiratory rate 22 bpm. What’s the initial management?
Correct Answer : D
Wait for 4h paracetamol level.
Q.147. A 70-year-old man is brought to the A & E department by his daughter following overdose of 10 paracetamol tablets. His wife died 2 weeks ago. His only relevant past medical history was treatment for depression 10 years ago. His paracetamol level is below the treatment level. What is the most appropriate initial management?
Correct Answer : A
Referral for social worker
Q.148. A 17-year-old lady presents to the A&E department after taking 45 tablets of paracetamol following an argument with her boyfriend. The incident happened 10 hours ago. What is the most appropriate next action?
Correct Answer : C
If >8–24h and suspicion of large overdose (>7.5g) on the side of caution and start N-acetylcysteine, stopping it if the level is below the treatment line and INR/ALT normal.
Q.149. A 3-year-old child has been brought with facial lacerations. On examination he has some cuts over his right cheek and under the eye. The GCS on initial evaluation is 15. What is the appropriate next investigation?
Correct Answer : B
Normal GCS makes intracranial lesions less likely. As there is a facial injury to exclude any facial bone fracture we can do a facial X-ray.
Q.150. A 24-year-old biker has been rescued after being trapped under rocks for almost 12 hours. He complains of reddish brown urine. His creatinine is 350umol/L and his urea is 15mmol/L. What is the most important step in the management of this patient?
Correct Answer : B
It is a case of rhabdomyolysis which is initially treated with IV NS.
Q.151. A man brought into the emergency department after being stabbed in the chest. Chest is bilaterally clear with muffled heart sounds. BP is 60/nil. Pulse is 120bpm. JVP raised. What is the most likely diagnosis?
Correct Answer : B
Chest is clear, so there is no pneumothorax or pleural effusion.
Muffled heart sound is due to fluid in pericardial space, low BP from reduced chamber expansion due to pericardial fluid’s pressure and restricted right heart expansion causes raised JVP.
The three classic signs (Beck's triad) that indicate cardiac tamponade are: Low pressure in the arteries (hypotension) Bulging (distended) neck veins. Muffled heart sounds.
Q.152. A young man is brought to the emergency department after an RTA. His GCS on initial evaluation is 6. What is the most appropriate next step?
Correct Answer : D
Secure airway
Q.153. A butcher stabbed accidently his groin. He bled so much that the towel was soaked in blood and BP 80/50mmHg, pulse 130bpm. What % of circulatory blood did he lose?
Correct Answer : C
30-40%
Classification:
Class 1 up to 15% of blood volume lost
Class 2 15%-30%
Class 3 30%-40%
Class 4 greater than 40%.
Q.154. A girl with history of allergies visited a friend’s farm. She got stridor, wheeze and erythematous rash. What is the most appropriate treatment?
Correct Answer : A
Presence of stridor and wheeze are suggestive of anaphylaxis and the treatment option is IM adrenaline.
Q.155. A 23-year-old man has been stabbed in the back and has SOB. The trachea is not deviated, he has engorged neck veins and absent breath sounds on the right. What is the most appropriate diagnosis?
Correct Answer : A
Tension pneumothorax.
Points in favour:
i) Stab wound in the back
ii) SOB
iii) Engorged neck vein
iv) Absent breath sound
Q.156. A 23-year-old male has a tonic clonic seizure whilst at college. His GCS is 12, BP 120/77mmHg, Heart rate 99bpm. What is the most appropriate investigation for his condition?
Correct Answer : C
Iit is also possible that he may have taken the drug, even though first we have to do serum glucose as low blood glucose can be very easily managed and it needs urgent management to save life. If it is excluded then we can look for other causes which may be not fatal in a short time.
Q.157. A young footballer has collapsed during a game. During initial evaluation: RR 14/min, Pulse 88bpm, BP 110/70mmHg. He seems to be sweating and muttering some incomprehensible words. What is the most important next step?
Correct Answer : C
Normal vital signs, sweating, and collapse during exercise give suspicion of hypoglycemia which is very easy to treat but fatal if left unrecognized. So the most important next step is to check the blood sugar
Q.158. A 47-year-old man met with a RTA. He has multiple injuries. Pelvic fracture is confirmed. He has not passed urine in the last 4 hours. What is the next appropriate management for this patient?
Correct Answer : B
In pelvic fracture there is the chance of urethral rupture and hence displacement of the urethral catheter is likely while trying to place it.
Q.159. A 27-year-old woman has hit her neck in an RTA without complains of tingling or motor loss. What is the next most appropriate investigation?
Correct Answer : B
As there is no neurological deficit we can exclude any fracture by X-ray first.
Diagonal x-ray means, oblique view of cervical spine. By this view, we can assess facet joint arthopathy. This isn’t related to RTA.
Q.160. A man has discharge from his left ear after a fight. Where is the discharge coming from?
Correct Answer : A
Probable fracture base of the skull.
Q.161. A 29-year-old male brought to emergency department in unconscious state. There is no significant past history. Which of the following should be done as the initial investigation?
Correct Answer : B
Blood glucose
Q.162. An old lady 72-year-old staying at a nursing home for a few years, a known Hypertensive on regular treatment presented with sudden dysphagia while eating with drooling of saliva and require urgent investigation. What would be your next step?
Correct Answer : C
Probably impacted food bolus (usually meat) which can be visualized and removed with the aid of endoscopy.
Q.163. A 34-year-old man after a car crash is in the emergency department and deteriorating. His GCS has fallen from 13 to 7. What is the most appropriate next step?
Correct Answer : D
Intubation. [ABC protocol]
Q.164. 34-year-old man was brought to the emergency department after a RTA (road traffic accident). BP 50/0mmHg and chest wall not moving symmetrically, RR (respiratory rate) is 34 bpm. What would be initial action?
Correct Answer : B
Intubation and ventilation [ABC protocol].
Q.165. A young man who was held by the police was punched while in custody. He is now cyanosed and unresponsive. What is the 1st thing you would do?
Correct Answer : B
Clear airway. [ABC protocol].
Q.166. A 16-year-old teenager was brought to the emergency department after being stabbed on the upper right side of his back. Erect CXR revealed homogenous opacity on the lower right lung, trachea was centrally placed. What is the most probable explanation for the X-ray findings?
Correct Answer : B
In blunt trauma there may be hemopneumothorax but in a sharp wound like stabbing there may occur only hemothorax.
Q.167. A 14-year-old boy fell and hit his head in the playground school. He didn’t lose consciousness. He has swelling and tenderness of the right cheek with a subconjuctival hemorrhage on his right eye. What is the most appropriate initial investigation
Correct Answer : D
There is no sign of basal fracture.
Panda sign is suggestive of basal fracture, not subconjunctival hemorrhage- unless posterior border of hemorrhage is not visible and no indication of CT scan (there are some criteria for CT scan which are not fulfilled here).
Q.168. A patient was admitted to the emergency department after a head injury. When examined on arrival his GCS was 15 and then at night his GCS deteriorated to 12. What investigation should be done?
Correct Answer : A
Initial GCS 15 followed by later GCS 12 are suggestive of intracranial hemorrhage. So the best investigation to be done is CT head.
Q.169. A 32-year-old miner is rescued after being trapped under a fallen rock for 4h. After applying a bladder catether, 15-20ml of reddish brown urine was obtained. HR 120bpm, SBP 100mmHg. What would be the next appropriate step?
Correct Answer : B
The diagnosis is rhabdomyolysis. So IV fluid is the next appropriate step.
Q.170. A 27-year-old man presents to the emergency department after an RTA (road traffic accident) where his foot was stuck under a truck for several hours. He now has swelling on the foot. Examination: foot tenderness, loss of sense in the space between the 3rd metatarsal and big toe and his dorsalis pedis is not felt. What is the most likely diagnosis?
Correct Answer : A
Acute compartment syndrome occurs after a traumatic injury such as a car crash.
The trauma causes severe high pressure in the compartment which results in insufficient blood supply to muscles and nerves.
Acute compartment syndrome is a medical emergency that requires surgery to correct.
If untreated, the lack of blood supply leads to permanent muscle and nerve damage and can result in the loss of function of the limb.
Q.171. A man presents with abdominal pain, vomiting, pulse 120bpm, BP 90/60mmHg and a rigid abdomen. His chest is clear. What is the immediate management?
Correct Answer : C
Urgent admission to ITU.
Q.172. A 34-year-old man was involved in a RTA and whilst in the ambulance his GCS deteriorated and RR increased from 30-48. What is the most appropriate management for this patient?
Correct Answer : C
100% oxygen
Q.173. A 24-year-old male was trying to move his wardrobe but it fell on his thigh. It was there for a very long time before someone was able to remove it. When he was seen in emergency department he had casts in his urine but no RBCs. Other inv showed hypocalcemia and high serum creatinine. What is the cause for his renal failure?
Correct Answer : B
The likely diagnosis is Rhabdomyolysis where renal failure occurs due to myoglobin (myoglobins are released after the breakdown of skeletal muscles).
Q.174. A young male met with an RTA and is suspected to have a femur fracture. His BP is 90/60mmHg. What is the next immediate action?
Correct Answer : B
IV fluids
Q.175. A 27-year-old patient met with a RTA. While the NGT (nasogastric tube) is passing, bowel sounds are heard in the chest. CXR shows NGT curled. What is the diagnosis?
Correct Answer : A
Diaphragm rupture
Q.176. A young man was knocked down during a fight in the waiting room of the emergency department. He is now unconscious and unresponsive. What is the 1st thing you would do?
Correct Answer : B
Put airway. [ABC Protocol]
Q.177. A 45-year-old heroin addict was involved in a car crash and is now paraplegic. During the 1st week of hospital stay he cried everyday because he couldn’t remember the accident. What is the most likely diagnosis?
Correct Answer : C
Organic brain damage
Q.178. A 34-year-old man after an RTA was brought to the emergency department. His BP is 50/0mmHg and chest wall with asymmetrical movement, RR (respiratory rate) 34bpm. What would be the initial action?
Correct Answer : B
Intubation and ventilation
Q.179. A 45-year-old female comes to the emergency department while having a generalized tonic clonic seizure and she has having difficulty breathing and is cyanosed. What is the treatment option for her?
Correct Answer : A
Breathing difficulty with cyanosis may indicate saliva, blood, foreign body (as denture), fall back of tongue etc. for which 1st action is to secure airway.
Q.180. An 87-year-old woman with a history of HTN has acute breathlessness. She has a RR (respiratory rate) of 32bpm, widespread lung crackles, pulse 120bpm, BP 160/90mmHg and elevated venous pressure. Her peripheral O2 saturation is 85%. What is the most appropriate initial management?
Correct Answer : D
100% oxygen.
The most appropriate initial management is E. 100% oxygen. Oxygen saturation is low. So we have to give oxygen initially.
Q.181. A patient is unresponsive and cyanosed. What is the most definitive 1st step in management?
Correct Answer : B
Check airway
Q.182. A 69-year-old male presented with sudden onset of dysphagia. He is neither able to swallow liquid nor solid, he recently had a denture fitting. What is the most probable diagnosis?
Correct Answer : A
Sudden onset of dysphagia to both liquid and solid and recent history of fitting denture suggests foreign body (denture) in oesophagus.
Q.183. A man got his hand caught in machinery at work. The fingers are swollen but the XR shows no fracture. What is the most appropriate management?
Correct Answer : D
Elevate in the high sling for 2 days.
Extravasetion of blood caused this swelling along with inflammation for which the best option is to elevate in the high sling for 2 days which will help improve symptoms by gravitational shift of fluid from hand.
Q.184. A 10-year-old boy has fallen from a tree and injured his right chest. He has pain and difficulty breathing. He is tachypenic and tender with an area of paradoxical chest wall movement on the right side. What is the most likely diagnosis?
Correct Answer : B
In flial chest multiple adjacent ribs are broken in multiple places, separating a segment, so a part of the chest wall moves independently. Pain, difficulty breathing, tachypnea and tender area of chest wall movement suggest flial chest.
Q.185. A 30-year-old woman has injured her left lower chest in a RTA (road traffic accident). She has BP 80/50 mmHg, pulse 120 bpm. Auscultation of chest bowel sounds present. What is the most likely diagnosis?
Correct Answer : A
Diaphragmatic rupture.
Bowel sound present on auscultation of the chest following RTA causing lower chest injury is suggestive of diaphragmatic rupture.
Q.186. A 40-year-old man has fallen off a roof. He is shocked and has chest pain. There is a delay between the radial and femoral pulse. His CXR shows widening of the mediastinum. What is the most likely diagnosis?
Correct Answer : D
Traumatic rupture of the aorta.
A traumatic aortic disruption is caused by a rapid acceleration (or deceleration) causing a tear in the aorta. Normally this is immediately fatal, but those who survive may show a widened mediastinum on CXR.
This can be confirmed with CT scan or angiography of the aorta and requires prompt surgical correction. Stable the haemodynamics and surgical correction.
Q.187. A 60-year-old man is brought to the emergency department in an agitated state. He is lashing out violently. Which drug in low dosage due to its relative lack of autonomic side effects is a drug of choice in the treatment of agitation in this patient?
Correct Answer : B
Haloperidol has autonomic side effects.
Q.188. A 35-year-old man skidded on a wet road while riding his motorbike at a speed of 70 mph. He has a large hematoma on temporal scalp, some bruises on chest wall and abdomen and a deformed thigh. GCS 11/15. High flow oxygen via mask given. Most immediate radiological investigation required during initial resuscitation phase?
Correct Answer : A
CXR may reveal possible chest structure trauma requiring urgent attention which may be life saving while brain lesions need time consuming CT and treatment and fracture femur may be managed taking more time.
Choose a Question
×Choose a Question
×
Trial Access Limit Reached
You’ve reached the limit of free content. Subscribe to continue learning without restrictions.